You are on page 1of 110

SIKKIM MANIPAL UNIVERSITY

INTERNAL ASSIGNMENT
MBA (Revised)
SEMESTER-4

NAME : VINOD PRASAD

ROLL NO. : 510817398

LEARNING CENTER CODE : 1736

DATE OF SUBMISSION : 15-12-2009


INDEX

SL.
SUBJECTS CODE PAGE N0
NO.

Strategic Management & Business


1. MB0036 03 – 29
Policy

2. International Business Management MB0037 30 – 49

Service Marketing And Customer


3. MK0006 50 – 68
Relationship Management
Advertising Management Sales
4. MK0007 69 – 80
Promotion

5. E-Marketing MK0008 81 – 97

6. International Marketing MK0009 98 – 114

Abhishek kumar yadav 520844307 Page 2


STRATEGIC MANAGEMENT & BUSINESS POLICY

(MB-0036)

ASSIGNMENT-1

Q.1 What do you understand by the term Strategy in the context of Business
Management and Policy? And what are the stages in the formulation of a
Strategy?

Ans. A strategy is an operational tool to achieve the goals, and thus, the corporate mission.
Strategies do not attempt to outline exactly how the enterprise is to accomplish its
objectives. A company may view downsizing as a strategy in a competitive market to
render cost-effective services. Thus, strategy provides a framework to guide thinking
and action. Strategies are very much useful in organisations for guiding, planning and
control.

Strategy is a way of life both at the macro as well as micro levels for everyone, whether
it is a nation or a company. To win over in a given complex situation, the organisations,
even trans-nationals adopt strategies. They make changes, if necessary, even to their
global strategies. An individual company may formulate its own strategy to bring out the
desired results. The eventual success of the organisation depends upon strategy
formulation and implementation.

The recently initiated moves such as globalisation, privatisation and liberalisation are
strategies to attain a globally competitive economy. Business management must focus
on following issues

a. Vision- For proper growth of the company.


b. Mission – What the company wants to achieve.
c. Goals – To achieve the above mission.
d. Objectives – To achieve the set goals

Abhishek kumar yadav 520844307 Page 3


e. Strategies – To achieve the above objectives
f. Policies – To control strategies
g. Programmes – For implementation of objectives
The above list outlines some of the key issues at every stage of action illustrating how:

a. The mission springs out from vision statements

b. Goals from the mission

c. Objectives from goals

d. Strategies from objectives

e. And programmes from objectives

It is the crux of the strategic management process. Strategy refers to the course of
action desired to achieve the objectives of the enterprise. Formulation, together with its
implementation, constitutes an integral part of the management activity. Managers use
strategies for different purposes such as to overcome competition, to increase sales, to
increase production, to motivate the employees to provide their best, and so on.
Implementation of a strategy is a crucial task as the formulation of it. There may be a lot
of resistance during the implementation process. It is necessary for the manager to be
very tactful to involve the members of his group in the formulation of strategy to facilitate
the implementation process.

Stages in Strategy Formulation and Implementation

a. Identification of mission and objectives

b. Environment scanning

c. Generic strategy alternatives

d. Strategy variations

e. Strategic choice

f. Allocation of resources and formulation of organisational structure

g. Formulation of plans, policies, programmes and administration

b. h) Evaluation and control

Q.2 What, in brief, are the types of Strategic Alliances and the purpose of each?
Supplement your answer with one real life example of each.

Ans. Strategic alliances constitute a viable alternative in addition to Strategic Alternatives.


Companies can develop alliances with the members of the strategic group and perform

Abhishek kumar yadav 520844307 Page 4


more effectively. These alliances may take any of the following forms. Following are the
different types of strategic Alliances:

1. Product and/or service alliance: Two or more companies may get together to
synergise their operations, seeking alliance for their products and/or services. A
manufacturing company may grant license to another company to produce its
products. The necessary market and product support, including technical know-how,
is provided as part of the alliance. Example :- Coca-cola initially provided such
support to Thums Up.

Two companies may jointly market their products which are complementary in
nature. Example :- 1) Chocolate companies more often tie up with toy companies. 2)
TV Channels tie-up with Cricket boards to telecast entire series of cricket matches
live.

Two companies, who come together in such an alliance, may produce a new product
altogether. Example :- Sony Music created a retail corner for itself in the ice-cream
parlours of Baskin-Robbins.

2. Promotional alliance: Two or more companies may come together to promote their
products and services. A company may agree to carry out a promotion campaign
during a given period for the products and/or services of another company.
Example :- The Cricket Board may permit Coke’s products to be displayed during
the cricket matches for a period of one year.

3. Logistic alliance: Here the focus is on developing or extending logistics support.


One company extends logistics support for another company’s products and
services. Example:- The outlets of Pizza Hut, Kolkata entered into a logistic alliance
with TDK Logistics Ltd., Hyderabad, to outsource the requirements of these outlets
from more than 30 vendors all over India – for instance, meat and eggs from
Hyderabad etc.

4. Pricing collaborations: Companies may join together for special pricing


collaborations. Example :- It is customary to find that hardware and software
companies in information technology sector offer each other price discounts.
Companies should be very careful in selecting strategic partners. The strategy
should be to select such a partner who has complementary strengths and who can
offset the present weaknesses.

Q.3 What is a Business Plan? What purpose does it serve?

Ans. A business plan is a detailed description of how an organization intends to produce,


market and sell a product or service. Whether the business is housing, commercial or

Abhishek kumar yadav 520844307 Page 5


some other enterprise, a good business plan describes to others and to your own board
of directors, management and staff the details of how you intend to operate and expand
your business.

A solid business plan describes who you are, what you do, how you will do it, your
capacity to do it, what financial resources are necessary to carry it out, and how you
intend to secure those resources. A well-written plan will serve as a guide through the
start-up phase of the business. It can also establish benchmarks to measure the
performance of your business venture in comparison with expectations and industry
standards. And most important, a good business plan will help to attract necessary
financing by demonstrating the feasibility of your venture and the level of thought and
professionalism you bring to the task.

A well-written plan will serve as a guide through the start-up phase of the business. It
can also establish benchmarks to measure the performance of your business venture in
comparison with expectations and industry standards. And most important, a good
business plan will help to attract necessary financing by demonstrating the feasibility of
your venture and the level of thought and professionalism you bring to the task. A good
business plan will help attract necessary financing by demonstrating the feasibility of
your venture and the level of thought and professionalism you bring to the task. A good
business plan serves the following purposes:

1. Revenue Generation – Your organization may hope to create a business that will
generate sufficient net income or profit to finance other programs, activities or
services provided by your organization.

2. Employment Creation – A new business venture may create job opportunities for
community residents or the constituency served by your organization.

3. Neighborhood Development Strategy – A new business venture might serve as


an anchor to a deteriorating neighborhood commercial area, attract additional
businesses to the area and fill a gap in existing retail services. You may need to find
a use for a vacant commercial property that blights a strategic area of your
neighborhood. Or your business might focus on the rehabilitation of dilapidated
single family homes in the community.

4. Establish Goals: Once you have identified goals for a new business venture, the
next step in the business planning process is to identify and select the right
business. Many organizations may find themselves starting at this point in the
process. Business opportunities may have been dropped at your doorstep.
Depending on the goals you have set, you might take several approaches to identify

Abhishek kumar yadav 520844307 Page 6


potential business opportunities.

5. Local Market Study: Whether your goal is to revitalize or fill space in a


neighborhood commercial district or to rehabilitate vacant housing stock, you should
conduct a local market study. A good market study will measure the level of existing
goods and services provided in the area, and assess the capacity of the area to
support existing and additional commercial or home-ownership activity. A bad or
insufficient market study could encourage your organization to pursue a business
destined to fail, with potentially disastrous results for the organization as a whole.
Through a market study you will be able to identify gaps in existing products and
services and unsatisfied demand for additional or expanded products and services.

6. Analysis of Local and Regional Industry Trends: Another method of investigating


potential business opportunities is to research local and regional business and
industry trends. You may be able to identify which business or industrial sectors are
growing or declining in your city, metropolitan area or region. The regional or
metropolitan area planning agency for your area is a good source of data on industry
trends.

7. Internal Capacity: The board, staff or membership of your organization may


possess knowledge and skills in a particular business sector or industry. Your
organization may wish to draw upon this internal expertise in selecting potential
business opportunities.

8. Internal Purchasing Needs / Collaborative Procurement: Perhaps,


theorganization frequently purchases a particular service or product. If nearby
affiliate organizations also use this service or product, this may present a business
opportunity. Examples of such products or services include printing or copying
services, travel services, transportation services, property management services,
office supplies, catering services, and other products.

Q.4 What is the chief purpose of a Business Continuity Plan and what are its
components for effective implementation. Explain in a sentence or two as to how
it is different from a Business Plan.

Ans. The Business Continuity Plan is a tool to allow organizations to consider the factors and
steps necessary to prepare for a crisis (disaster or emergency) so that it can manage
and survive the crisis and take all appropriate actions to help ensure the organization’s
continued viability. The advisory portion of the plan is divided into two parts:

 Planning process: It provides step-by-step Business Continuity Plan preparation


and activation guidance, including readiness, prevention, response, and recovery/

Abhishek kumar yadav 520844307 Page 7


resumption.

 Implementation and maintenance: It gives the details of tasks required for the
Business Continuity Plan to be maintained as a living document, changing and
growing with the organization and remaining relevant and executable.

The purpose of the business continuity plan is to prepare to face the unthinkable
situations that may threaten an organization’s future. This new challenge goes beyond
the mere emergency response plan or disaster management activities that we
previously employed. Organizations now must engage in a comprehensive process best
described generically as Business Continuity. It is no longer enough to draft a response
plan that anticipates naturally, accidentally, or intentionally caused disaster or
emergency scenarios.

Today’s threats require the creation of an on-going, interactive process that serves to
assure the continuation of an organization’s core activities before, during, and most
importantly, after a major crisis event.

In the simplest of terms, it is good business for a company to secure its assets. CEOs
and shareholders must be prepared to budget for and secure the necessary resources
to make this happen. It is necessary that an appropriate administrative structure be put
in place to effectively deal with crisis management.

Following steps are required to fulfilled for effective implementation of the business
continuity plan:

1. Educate and Train: The BCP is only as valuable as the knowledge that others
have of it. Education and training are necessary components of the BCP process.
They require a time commitment from the Crisis Management Team, the Response
Teams, and the general employee population.

2. Educate and Train Teams: The Crisis Management and Response Teams should
be educated about their responsibilities and duties. Check lists of critical actions
and information to be gathered are valuable tools in the education and response
processes.

3. Educate and Train All Personnel: All personnel should be trained to perform their
individual responsibilities in case of a crisis. Such training could include procedures
for evacuation, shelter-in-place, check-in processes to account for employees,
arrangements at alternate worksites, and the handling of media inquiries by the
company.

Abhishek kumar yadav 520844307 Page 8


4. Review of BCP: The BCP should be regularly reviewed and evaluated. Reviews
should occur according to a pre-determined schedule, and documentation of the
review should be maintained as necessary. The following factors can trigger a
review and should otherwise be examined once a review is scheduled:

 Risk Assessment
 Sector/Industry Trends
 Regulatory Requirements
 Event Experience
 Test/Exercise Results
5. Maintenance of BCP: Regular maintenance of the BCP cannot be
overemphasized. Clear responsibility for BCP maintenance should be assigned.
Maintenance can be either planned or unplanned and should reflect changes in the
operation of the organization that will affect the BCP.

Difference between a Business plan & Business continuity Plan

a. A Business plan is a detailed description of how an organisation intends to


produce, market and sale a product or service. A Business continuity plan is an
ongoing process supported by senior management and funded to ensure that the
necessary steps are taken to identify the impact of potential losses, maintain
viable recovery strategies and plans, and ensure the continuity of operations
through personnel training, plan ,testing and maintenance.

b. A Business continuity plan is a tool which allows organisations to consider the


factors and steps necessary to prepare for a crisis.(disaster or emergency).
Whereas a business plan is not prepared for such type of disaster or emergency.

c. In a business continuity plan, a necessary Administrative structure is put in place


to effectively deal with crisis management, whereas,in a business plan, no such
administrative structure is available.

Q.5 Take any three examples of the components of a Decision Support System and
explain how they help decision making

Ans. Following are the three components of a Decision Support System

1. Annual Budget: It is really a business plan. The budget allocates amounts of


money to every activity and/or department of the firm. As time passes, the actual
expenditures are compared to the budget in a feedback loop. During the year, or at
the end of the fiscal year, the firm generates its financial statements: the income

Abhishek kumar yadav 520844307 Page 9


statement, the balance sheet, the cash flow statement. When putting together,
these four documents are the formal edifice of the firm’s finances. However, they can
not serve as day-to-day guides to the General Manager.

2. Daily Financial Statements: The Manager should have access to continuously


updated statements of income, cash flow, and a balance sheet. The most important
statement is that of the cash flow. The manager should be able to know, at each and
every stage, what his real cash situation is – as opposed to the theoretical cash
situation which includes accounts payable and account receivable in the form of
expenses and income.

3. The Daily Ratios Report: This is the most important part of the decision support
system. It enables the Manager to instantly analyse dozens of important aspects of
the functioning of his company. It allows him to compare the behaviour of these
parameters to historical data and to simulate the future functioning of his company
under different scenarios. It also allows him to compare the performance of his
company to the performance of his competitors, other firms in his branch and to the
overall performance of the industry that he is operating in.

The Manager can review these financial and production ratios. Where there is a
strong deviation from historical patterns, or where the ratios warn about problems in
the future – management intervention may be required.

Examples of the Ratios to be Included in the Decision System

 SUE measure – deviation of actual profits from expected profits

 ROE – the return on the adjusted equity capital

 Debt to equity ratios

 ROA – the return on the assets

 The financial average

 ROS – the profit margin on the sales

 ATO – asset turnover, how efficiently assets are used

 Tax burden and interest burden ratios


 Compounded leverage
 Sales to fixed assets ratios

 Inventory turnover ratios


 Days receivable and days payable

Abhishek kumar yadav 520844307 Page 10


 Current ratio, quick ratio, interest coverage ratio and other liquidity and coverage
ratios
 Valuation price ratios
 And many others
A decision system has great impact on the profits of the company. It forces the
management to rationalize the depreciation, inventory and inflation policies. It warns the
management against impending crises and problems in the company. It specially helps
in following areas:

a. The management knows exactly how much credit it could take, for how long (for
which maturities) and in which interest rate. It has been proven that without proper
feedback, managers tend to take too much credit and burden the cash flow of their
companies.

b. A decision system allows for careful financial planning and tax planning. Profits go
up, non cash outlays are controlled, tax liabilities are minimized and cash flows are
maintained positive throughout.

The decision system is an integral part of financial management in the West. It is


completely compatible with western accounting methods and derives all the data that it
needs from information extant in the company.

So, the establishment of a decision system does not hinder the functioning of the
company in any way and does not interfere with the authority and functioning of the
financial department, but infact helps the manager to take quick decisions and make
profit to the company.

Q.6 Name and explain any three ways in which a Company’s CSR can be expressed

Ans. CSR is “a concept whereby companies integrate social and environmental concerns in
their business operations and in their interaction with their stakeholders on a voluntary
basis” as they are increasingly aware that responsible behaviour leads to sustainable
business success.

CSR is also about managing change at company level in a socially responsible manner.
This happens when a company seeks to set the trade-offs between the requirements
and the needs of the various stakeholders into a balance, which is acceptable to all
parties. If companies succeed in managing change in a socially responsible manner,
this will have a positive impact at the macro-economic level.

Following are the different ways in which company's CSR can be expressed.

Abhishek kumar yadav 520844307 Page 11


1. Employment and Social Affairs Policy

Within a business CSR relates to quality employment, life-long learning, information,


consultation and participation of workers, equal opportunities, integration of people with
disabilities anticipation of industrial change and restructuring. Social dialogue is seen as
a powerful instrument to address employment-related issues.

Employment and social policy integrates the principles of CSR, in particular, through the
European Employment Strategy, an initiative on socially responsible restructuring, the
European Social Inclusion Strategy, initiatives to promote equality and diversity in the
workplace, the EU Disability Strategy and the Health and Safety Strategy.

In its document "Anticipating and managing change: a dynamic approach to the social
aspects of corporate restructuring", the Commission has stressed that properly taking
into account and addressing the social impact of restructuring contributes to its
acceptance and to enhance its positive potential. The Commission has called upon the
social partners to give their opinion in relation to the usefulness of establishing at
Community level a number of principles for action, which would support business good
practice in restructuring situations.

Deeply rooted societal changes such as increasing participation of women in the labour
market should be reflected in CSR, adapting structural changes and changing the work
environment in order to create more balanced conditions for both genders
acknowledging the valuable contribution of women as strategies which will benefit the
society as well as the enterprise itself.

2. Enterprise policy

Only competitive and profitable enterprises are able to make a long-term contribution to
sustainable development by generating wealth and jobs without compromising the
social and environmental needs of society. In fact, only profitable firms are sustainable
and have better chances to adopt/develop responsible practices.

The role of enterprise policy is to help create a business environment, which supports
the Lisbon objective of becoming the world’s most dynamic knowledge-driven economy,
supports entrepreneurship and a sustainable economic growth. Its objective is to ensure
a balanced approach to sustainable development, which maximises synergies between
its economic, social and environmental dimensions.

3. Consumer Policy

CSR has partly evolved in response to consumer demands and expectations.


Consumers, in their purchasing behaviour, increasingly require information and

Abhishek kumar yadav 520844307 Page 12


reassurance that their wider interests, such as environmental and social concerns, are
being taken into account.

Consumers and their representative organisations have an important role to play in the
evolution of CSR. If CSR is therefore to continue to serve its purpose, strong lines of
communication between enterprises and consumers need to be created.

STRATEGIC MANAGEMENT & BUSINESS POLICY

(MB-0036)

ASSIGNMENT-2

Q.1 Having formulated a Business Strategy, what are the steps in its implementation?
Explain each in a sentence or two.

Ans. A strategy is an operational tool to achieve the goals, and thus, the corporate mission.
Strategies do not attempt to outline exactly how the enterprise is to accomplish its
objectives. A company may view downsizing as a strategy in a competitive market to
render cost-effective services. Thus, strategy provides a framework to guide thinking
and action. Strategies are very much useful in organizations for guiding, planning and
control.

Strategy is a way of life both at the macro as well as micro levels for everyone, whether
it is a nation or a company. To win over in a given complex situation, the organizations,
even trans-nationals adopt strategies. They make changes, if necessary, even to their
global strategies. An individual company may formulate its own strategy to bring out the
desired results. The eventual success of the organization depends upon strategy
formulation and implementation.

Strategy refers to the course of action desired to achieve the objectives of the
enterprise. Formulation, together with its implementation, constitutes an integral part of
the management activity. Managers use strategies for different purposes such as to
overcome competition, to increase sales, to increase production, to motivate the
employees to provide their best, and so on. Implementation of a strategy is a crucial
task as the formulation of it. There may be a lot of resistance during the implementation
process. It is necessary for the manager to be very tactful to involve the members of his
group in the formulation of strategy to facilitate the implementation process.

Abhishek kumar yadav 520844307 Page 13


Following are the steps for implementing the business strategy

1. Identification of mission and objectives


2. Environment scanning
3. Generic strategy alternatives
4. Strategy variations
5. Strategic choice
6. Allocation of resources and formulation of organizational structure
7. Formulation of plans, policies, programmes and administration
8. Evaluation and control.
After formulating a business strategy, the process of strategy implementation calls for
an integrated set of choices and activities. These include

1. Allocating Resources: A good strategy with effective implementation has a higher


probability of success. There source allocation decisions, such as, which department
is sanctioned how much of money and resources, in the name of the budget, and so
on – set the operative strategy of the firm. Budgets are formulated after a series of
negotiations across different levels in the organization. Budgets may be of different
types: corporate budgets, capital budgets, departmental budgets, sales budgets,
expense budgets, and others.

2. Organising: An effective co-ordination and efficient division of labour requires an


appropriate organizational structure. The best structure is one, which fits into the
organizational environment. Also its internal characteristics should give rise to an
effective strategy. Appropriate changes in the organization structure may be
initiated to ensure strategic implementation of the proposed strategy. Effective
strategic management practices suggest that organization structure should also
change if the strategy changes or if the organization experiences any bottlenecks in
this regard

3. Formulation Of Policies, Plans, Programmes And Administration: The


resources allocated are said to be well-utilized only when they are well-monitored.
For this purpose, it is essential:

 To develop policies and plans.

 To assign and reassign leaders the tasks and decisions to support the
chosen strategy.

 To provide a conducive environment in the organization through proper

Abhishek kumar yadav 520844307 Page 14


administration to achieve the given objectives directly and indirectly.

The implementation of plans and policies is designed in accordance with the strategy
chosen. The firm creates plans and policies to guide managerial performance, and
these make the chosen strategies work. The corporate success lies ultimately in the
ability to convert corporate strategy into plans and policies that are compatible and
workable.

4. Evaluation And Control Of Strategy: Evaluation is the last phase of the strategic
management process. It is at this stage that the success of the programmes can be
assessed. There should be a built-in mechanism to examine the deviations and
initiate corrections as and when required. This assures that the chosen strategies
will be implemented properly.

The control process requires identifying a set of parameters for evaluating and
measuring the performance at the individual level and also at the department level.
The performance has to be evaluated to identify deviations and take corrective
action. The control and evaluation take place not only at the SBU level but also at the
corporate level. This process may involve the participation of all the executives at all
levels. Corrective actions are required wherever the evaluation reveals deviations
between the actual performance and the projected one, over a given period of time.

Q.2 How do we cope with crises and how do we use the Business Continuity Plan to
manage and recover from crisis?

Ans. The Business Continuity Guideline is a tool to allow organizations to consider the
factors and steps necessary to prepare for a crisis (disaster or emergency) so that it can
manage and survive the crisis and take all appropriate actions to help ensure the
organization’s continued viability.

The advisory portion of the guideline which helps us to cope with the crisis and to
manage and recover from the crisis is divided into two parts:

1. The Planning Process - provides step-by-step Business Continuity Plan


preparation and activation guidance, including readiness, prevention, response, and
recovery/ resumption. The following steps are important:

i. Assign Accountability
ii. Perform Risk Assessment
iii. Conduct Business Impact Analysis (BIA)
iv. Agree on Strategic Plans
v. Crisis Management and Response Team Development

Abhishek kumar yadav 520844307 Page 15


vi. Mitigation Strategies
vii. Avoidance, Deterrence and Detection
viii. Potential Crisis Recognition
ix. Communications
x. Resource Management
xi. Financial Support
xii. Financial Issues and Insurance
xiii. Suppliers/Service Providers
xiv. Mutual Aid Agreements
xv. Damage and Impact Assessment
xvi. Resumption of Critical and Remaining Processes
xvii. Return to Normal Operations
2. Successful Implementation And Maintenance - details those tasks required for
the Business Continuity Plan to be maintained as a living document, changing and
growing with the organization and remaining relevant and executable. The various
steps involved are:

i. Educate and Train


ii. Test the BCP
iii. Develop BCP Review Schedule
iv. Develop BCP Maintenance Schedule
In order to use the Business Continuity Plan to manage and recover from the crisis, the
following checklists on the basis of the above advisory portion of the guideline needs to
be checked and necessary requirements as per the same, needs to be kept in ready
mode for immediate applications.

Accountability:

1. Does your organization’s policy include a definition of crisis?


2. Has the person responsible for critical systems and business processes been
identified?
3. Has a BCP Team been appointed, and does it include senior business function
leaders?
4. Has the BCP been communicated throughout the organization?
5. Has a person been assigned with the responsibility to update the BCP?
Risk Assessment

Abhishek kumar yadav 520844307 Page 16


1. Has your organization conducted a Risk Assessment?
2. Have the types of risks that may impact your organization been identified and
analyzed?
3. Has the likelihood for each type of risk been rated?
Business Impact Analysis

1. Have the critical business processes been identified?


2. Have the business processes been ranked (low, medium, high)?
3. If a crisis were to happen, has the impact, in terms of human and financial costs,
been assessed?
4. Have the maximum allowable outage and recovery time objectives been
determined?
5. Has the length of time your organization’s business processes could be non-
functional been determined?
6. Have the recovery time objectives been identified?
7. Have the resources required for resumption and recovery been identified?
Strategic Plans

1. Have methods to mitigate the risks identified in the Business Impact Analysis and
Risk Assessment been identified?

2. Have plans and procedures to respond to any incident been developed?

3. Have strategies that address short and long term business interruptions been
selected?

4. Are the strategies attainable, tested, and cost effective?

Compliance with Corporate Policy & Mitigation Strategies

1. Have compliance audits been conducted to enforce BCP policy and procedures?

2. Have the systems and resources that will contribute to the mitigation process been
identified, including personnel, facilities, technology, and equipment?

3. Have the systems and resources been monitored to ensure they will be available
when needed?

Avoidance, Deterrence, and Detection

1. Are employees motivated to be responsible for avoidance and deterrence and

Abhishek kumar yadav 520844307 Page 17


detection?

2. Have facility security programs to support avoidance and deterrence and detection
been established?

3. Have operational policy and procedures to protect the facilities been developed?

4. Is it ensured that sufficient physical security systems and planning are in place to
protect the facility?

Response: Potential Crisis Recognition and Team Notification

1. Will the response program recognize when a crisis occurs and provide some level
of response?

2. Have the danger signals been identified that indicate a crisis is imminent?

3. Have personnel been trained to observe warning signs of an imminent crisis?

4. Has a notification system been put in place, including redundant systems?

5. Is the notification contact list complete and up to date?

Assess the Situation

1. Has an assessment process to address the severity and impact of the crisis been
developed?

2. Has the responsibility for declaring a crisis, with first and second alternates, been
assigned?

Declare a Crisis

1. Have the criteria been established for when a crisis should be declared?

2. Has the responsibility for declaring a crisis been clearly defined and assigned?

3. Has an alert network for BCP Team members and employees been established?

4. Is it ensured that there is an alternate means of warning if the alert network fails?

5. Have the activities that will be implemented in event of a crisis been identified,
including notification, evacuation, relocation, alternate site activation, team
deployment, operational changes, etc?

Execute the Plan

Abhishek kumar yadav 520844307 Page 18


1. Has consideration been given to developing the BCP around a ‘‘worst case
scenario?’’

2. Has the BCP been prioritized to save lives, protect assets, restore critical business
processes and systems, reduce the length of the interruption, protect reputation,
control media coverage, and maintain customer relations?

3. Have the severity of the crisis and the appropriate response been determined?

Communications

1. Has a crisis communications strategy been developed?

2. Are communications timely, honest, and objective?

3. Are communications with all employees occurring at approximately the same time?

4. Are regular updates provided, including notification of when the next update will be
issued?

5. Has a primary spokesperson and back-up spokespersons been designated who


will manage and disseminate crisis communications to the media and others?

Resource Management – Logistics

1. Has a designated Crisis Management Center been identified, and does it have
necessary life support functions, including uninterruptible power supply and
communications equipment?

2. Have alternate worksites for business resumption and recovery been identified?

3. Have critical and vital records been stored at an offsite storage facility?

4. How long can each business function operate effectively without normal data input
storage processes?

5. What must be done to restore data to the same previous point in time within the
recovery time objective?

6. Can any alternate data storage processes be used, after the initial data recovery,
to speed the forward recovery to the present time?

Resource Management – Financial Issues and Insurance, Transportation,


Suppliers/Service Providers, and Mutual Aid

Abhishek kumar yadav 520844307 Page 19


1. Has the appropriate insurance coverage been identified and obtained?

2. Are cash and credit available to the BCP Team?

3. Have transportation alternatives been arranged in advance?

4. Have critical vendor and service provider agreements been established?

5. Have mutual aid agreements been established?

6. If so, are they legally sound, properly documented, and understood by all parties?

Recovery and resumption

1. Damage and Impact Assessment, Process Resumption, and Return to Normal


Operations

2. Has a damage assessment been performed as soon as possible?

3. Has the Damage Assessment Team been mobilized to the site?

4. Has business process recovery been prioritized to recover the most critical
business processes first?

5. Is the schedule of the processes to be restored in accordance with the prioritization


schedule?

6. Is there documentation of when the processes were resumed?

7. Has the organization returned to normal operations?

8. Has the decision to return to normal operations been documented and


communicated?

The preparation and readiness of the resources as per the above checklists on the
basis of the Business Continuity Plan helps us to manage and recover from the crisis in
a successful manner.

Q.3 What are the main components of Business Plan, explaining the role of each in
the Plan?

Ans. A business plan is a detailed description of how an organization intends to produce,


market and sell a product or service. A solid business plan describes who you are, what
you do, how you will do it, your capacity to do it, what financial resources are necessary
to carry it out, and how you intend to secure those resources. A well-written plan will
serve as a guide through the start-up phase of the business. It can also establish

Abhishek kumar yadav 520844307 Page 20


benchmarks to measure the performance of your business venture in comparison with
expectations and industry standards. And most important, a good business plan will
help to attract necessary financing by demonstrating the feasibility of your venture and
the level of thought and professionalism you bring to the task.

A good business plan will help attract necessary financing by demonstrating the
feasibility of your venture and the level of thought and professionalism you bring to the
task.

A business plan should contain the following components:

1. Executive Summary: The purpose of this component is to provide direction and


help you establish your goals as clearly as possible. With this clearly spelled out,
both management and employees can focus their efforts. Also it will help with
investors and bankers in that they can clearly understand what your. It should
include the following information.

 A history of the company (if you're already in business) or a history of the idea
development.

 A description of the products or services you plan to develop and sell.

2. Company and Product Description: In this component one should provide type of
business, reason of creating business unit, referencing the goal and attach
references of business plan.

 Product or Service-Focus on the feature that distinguishes from rest of market


and what will attract consumers to your product or service.

 Price-Provide realistic price estimate and discuss the rationale behind that price.
Describe where this price positions you in the market place: at the high end, low
end or in the middle of the existing range for a similar product or service.

 Place-Describe the location and its advantages where you will produce or
distribute your product or provide your service.

 Customers-Providing detail description of the customer base depending upon


their characteristics i.e. demographics, geography and socio economic status.
Provide statistical data to describe the size of target customer base and its
growth potential.

 Competition-Explain the reason for entering in the new venture and what

Abhishek kumar yadav 520844307 Page 21


differentiate your proposed venture from existing business.

 Sales Projections-Present an estimate based on size of your market, the


characteristic of your customers and the share of market you will gain over your
competition.

3. Market Description: A key composition of the operation of your business will be


your sales and marketing strategy, so one must describe how you will inform your
target market about your product or service and how you will convince customer to
purchase.

 Production Description:-Describe the steps from raw material to finished goods,


packaged and ready for distribution and sale. Avoid using industry jargon to
describe the production process.

 Staffing:-Describe the type of staff, their qualification, skill, salaries, other benefit
and how they will be recruited.

4. Equipment and Material: In this component one must describe the special type of
machines and materials required to purchase, their cost and source.

 Facility:-Facilities and other infrastructure required for the company, specialized


features if any are to be described.

 Market Description:-Strategy for locating your target market, informing or


educating customers about your product or service and convincing them to buy
it. Discuss the product‘s feature you plan to emphasize to gain the attention of
your target market.

5. Operations: In this component company must highlight the senior manager


responsible for overseeing the start-up and operation of your business, their
background and their responsibility in the business. Ownership- Potential investors
and lenders will be interested in ownership stake of the board of directors and also in
what portion of the company‘s equity is available. An investor or banker must know
this because each form has certain legal and tax implications. The typical forms are:

• Sole proprietor.
• Partnership.
• Limited Liability Company
• Corporation.
• Sub-Chapter S Corporation

Abhishek kumar yadav 520844307 Page 22


6. Management and ownership: Providing a brief description of the different financial
statements of the business to be attached such as ….

 Startup budget-Describe the initial expenses which may incur to get your
business up and running.

 Cash flow projection-Provide month to month schedule of estimated cash inflow


and outflows from the business for the first year. This include how much money
and when.

 Income statement-Prepare a multiyear statement of projected revenue,


expenses, capital expenditure and cost of goods sold. This should be supported
by documents of growth pattern of similar companies or studies that forecast an
industry wise forecast.

 Balance sheet-Provide a copy of balance sheet of sponsoring organization.

7. Financial information and timeline:

 Capital Requirement- Describe the Requirement of type of capital for financing


and repayment period, amount of expected return and explain any commitment
or investment that you may have already secured. Types of requirement of funds
are.

 Seed Capital

 Fixed asset financing

 Working Capital

 Initial Start-up timeline-Provide a timeline of task and events necessary to get


your business operational. Be sure to record stage as on today also. Set realistic
dead line to capacity to complete these tasks. There may be several reasons to
delay the project which are in not ones hand but explain in details.

 describe any asset that will be allocated to the start-up of the business.

8. Risks and their mitigation: Although it is impossible to know exactly what will go
wrong in staring and running your business plan, thinking upon different challenges
will strengthen plan.

Q.4 Explain the concept, need for and importance of a Decision Support System.

Ans. Concept: Decision support systems constitute a class of computer-based information

Abhishek kumar yadav 520844307 Page 23


systems including knowledge-based systems that support decision-making activities.

It is also explained as a class of information systems (including but not limited to


computerized systems) that support business and organizational decision-making
activities. A properly designed DSS is an interactive software-based system intended to
help decision makers compile useful information from a combination of raw data,
documents, personal knowledge, or business models to identify and solve problems and
make decisions.

Typical information that a decision support application might gather and present are:

• an inventory of all of your current information assets (including legacy and


relational data sources, cubes, data warehouses, and data marts),

• comparative sales figures between one week and the next,

• projected revenue figures based on new product sales assumptions.

Need: Many companies in developing countries have a very detailed reporting system
going down to the level of a single product, a single supplier, a single day. However,
these reports – which are normally provided to the General Manager – should not be
used by them at all. They are too detailed and, thus, tend to obscure the true picture. A
General Manager must have a bird’s eye view of his company. He must be alerted to
unusual happenings, disturbing financial data and other irregularities.

As things stand now, the following phenomena could happen:

• That the management will highly leverage the company by assuming excessive
debts burdening the cash flow of the company.

• That a false Profit and Loss (PNL) picture will emerge – both on the single product
level – and generally. This could lead to wrong decision-making, based on wrong
data.

• That the company will pay excessive taxes on its earnings.

• That the inventory will not be fully controlled and appraised centrally.

• That the wrong cash flow picture will distort the decisions of the management and
lead to wrong (even to dangerous) decisions.

Importance: A decision system has great impact on the profits of the company. It
forces the management to rationalize the depreciation, inventory and inflation policies. It
warns the management against impending crises and problems in the company. It

Abhishek kumar yadav 520844307 Page 24


specially helps in following areas:

 The management knows exactly how much credit it could take, for how long (for
which maturities) and in which interest rate. It has been proven that without proper
feedback, managers tend to take too much credit and burden the cash flow of their
companies.

 A decision system allows for careful financial planning and tax planning. Profits go
up, non cash outlays are controlled, tax liabilities are minimized and cash flows are
maintained positive throughout.

The decision system is an integral part of financial management in the West. It is


completely compatible with western accounting methods and derives all the data that it
needs from information extant in the company.

So, the establishment of a decision system does not hinder the functioning of the
company in any way and does not interfere with the authority and functioning of the
financial department.

Q.5 Explain the importance of any five aspects of a Licensing Agreement that you will
look for when negotiating the right to use an Intellectual Property

Ans. A licence is a grant of permission made by the patent owner to another to exercise any
specified rights as agreed. Licensing is a good way for an owner to benefit from their
work as they retain ownership of the patented invention while granting permission to
others to use it and gaining benefits, such as financial royalties, from that use. However,
it normally requires the owner of the invention to invest time and resources in monitoring
the licensed use, and in maintaining and enforcing the underlying IP right.

A licence is merely the grant of permission to undertake some of the actions covered by
intellectual property rights, and the patent holder retains ownership and control of the
basic patent.

Licences are often limited to specific rights, territories and time periods. If an inventor
owns patents on the same invention in five different countries, they could assign (or sell)
these patents to five different owners in each of those countries. Portions of a patent
right can also be assigned – so that in order to finance your invention, you might choose
to sell a half-share to a commercial partner.

A lot of aspects are to be taken care of very carefully while negotiating or making a
licensing agreement to give right to use an intellectual property.

Some of the important aspects of a Licensing Agreement that we will look for when

Abhishek kumar yadav 520844307 Page 25


negotiating the right to use an Intellectual Property are explained below:

1. The Parties To The Licence: The ‘parties’ to the licence are the entities (individual
people, companies or institutions) that are bound by the licence as a legal contract.
They normally sign (or ‘execute’) the licence to confirm in a legally clear-cut way that
they agree to comply with its terms. The parties can be individual persons, but they
are normally legal entities such as a research institution, a university or a company.
It is important to ensure that the licence is signed in a way that is legally binding, and
by a person authorized to sign for their institution.

In some cases, the licence is not directly signed (think of the ‘I Agree’ block which
you are often asked to click when installing a software package on a computer), and
in the national law of many countries, contractual obligations can arise even without
a formal signed document.

2. Licensor And Licensee: The person granting the licence – the ‘licensor’ – typically
holds rights to technology, biological material, IP rights, know-how, or other
information. The person receiving the licence – the ‘licensee’ – is the party which is
seeking to use or exploit that material.

The basic idea of the licence is that the person granting the licence (the ‘licensor’) is
giving another person (the ‘licensee’) the right to do something that they could
otherwise prevent them from doing, in exchange for some kind of benefit (this may
be financial, but may be other forms of benefit). So the licensor might permit the
licensee to use a technology that is covered by a patent – and if there was no
license, the licensor could take legal action under the patent to prevent this use of
the technology.

3. Sole, Exclusive Or Non-Exclusive Licences: The choice taken among these


options is very important.

 An exclusive licence means that the licensor agrees not to grant another
licence to any other party, and agrees not to use the licensed rights (in other
words, the licensor cannot become a competing user of the licensed
technology).

 A sole licence means that the licensor grants a licence to only one licensee,
the sole licensee, but the licensor retains the right to use the technology itself.

 Under a non-exclusive licence, the licensor grants the licensee a right to use
the technology, but the licensor can still give the same rights to other

Abhishek kumar yadav 520844307 Page 26


licensees.

The kind of license granted will depend on several factors, and will do much to
influence the pattern of use, and the scale of royalties or other payments, made by
the licensee.

4. Sub-Licences: A ‘sub-licence’ is a further licence, when the licensee of the original


licence itself grants a licence to a third party. The sub-licence may extend to some or
all of the rights granted under the original licence. The original licence may need to
make clear whether sub-licences can be granted, and if so, to who, and on what
terms or conditions. There may be issues such as protecting the confidentiality of
licensed material, liability for use of the technology, or the interests of the licensor in
granting direct licenses to the same third parties.

A sub-licensing program may be of considerable benefit to the licensor, as it will


increase the scope of use of the licensed technology, and may be an effective way
of exporting the technology to new overseas markets. The original licensor might
retain an entitlement to a share of any royalties or other payments paid by the sub-
licensee. The original licensor might retain the right to investigate and approve the
eligibility of a sub-licensee, especially if the sub-licensee has no direct relationship
with the original licensee.

5. Diligence And Milestones: The licensee may be relying on the license as the
principal mechanism for recovering its investment in research and for deriving
benefits from the patent or other IP. So it may be important to the licensor to ensure
that the licensee does everything they can to develop and commercialize the
licensed IP. If a licensee gains an exclusive licence, subject to a royalty payment on
profits, and then decides to shelve the technology for several years because its
immediate interests lie elsewhere, then the whole value of the IP is lost to the
licensor. So licenses will frequently include obligations on the licensee to develop
and apply the licensed technology diligently and to meet specific deadlines. Where
possible, certain defined points or milestones should be identified – possibly based
on the business plan originally proposed by the licensee. The license could require
the licensee to bring the product to the market as soon as practicable, and to
continue to make the product available to the public on reasonable terms. These
obligations may also be built into sub-licenses granted under the licence.

6. Payments And Pricing: A licence will normally involve a valuable ‘consideration’ –


something of value which is given in exchange for the right to use this technology. It
need not be financial consideration – it could be a right in exchange to use another
technology (a ‘cross-licence’), or valuable access to other facilities or resources.

Abhishek kumar yadav 520844307 Page 27


There are many potential models for payment. It is always difficult to establish a
value for intellectual property, even more so if it relates to unproven technology that
will require a licensee to take a considerable commercial risk. The options boil down
to lump sum payments, and royalties based on the extent of use of the technology. It
is not unusual to see a mixture of both.

Q.6 What is Corporate Social Responsibility? Why is it becoming increasingly


relevant in today’s Business?

Ans. Corporate social responsibility (CSR), also known as corporate responsibility,


corporate citizenship, responsible business, sustainable responsible business
(SRB), or corporate social performance, is a form of corporate self-regulation
integrated into a business model. Ideally, CSR policy would function as a built-in, self-
regulating mechanism whereby business would monitor and ensure its adherence to
law, ethical standards, and international norms. Business would embrace responsibility
for the impact of their activities on the environment, consumers, employees,
communities, stakeholders and all other members of the public sphere. Furthermore,
business would proactively promote the public interest by encouraging community
growth and development, and voluntarily eliminating practices that harm the public
sphere, regardless of legality.

Essentially, CSR is the deliberate inclusion of public interest into corporate


decision-making, and the honoring of a triple bottom line: People, Planet and
Profit.

The practice of CSR is subject to much debate and criticism. Proponents argue that
there is a strong business case for CSR, in that corporations benefit in multiple ways by
operating with a perspective broader and longer than their own immediate, short-term
profits. Critics argue that CSR distracts from the fundamental economic role of
businesses; others argue that it is nothing more than superficial window-dressing;
others yet argue that it is an attempt to pre-empt the role of governments as a watchdog
over powerful multinational corporations. Corporate Social Responsibility has been
redefined throughout the years. However, it essentially is titled to aid to an
organization's mission as well as a guide to what the company stands for and will
uphold to its consumers.

Relevancy In Today’s Business:

Business ethics is one of the forms of applied ethics that examines ethical principles
and moral or ethical problems that can arise in a business environment.

In the increasingly conscience-focused marketplaces of the 21st century, the demand


for more ethical business processes and actions (known as ethicism) is increasing.

Abhishek kumar yadav 520844307 Page 28


Simultaneously, pressure is applied on industry to improve business ethics through new
public initiatives and laws (e.g. higher UK road tax for higher-emission vehicles).

Business ethics can be both a normative and a descriptive discipline. As a corporate


practice and a career specialization, the field is primarily normative. In academia,
descriptive approaches are also taken. The range and quantity of business ethical
issues reflects the degree to which business is perceived to be at odds with non-
economic social values. Historically, interest in business ethics accelerated dramatically
during the 1980s and 1990s, both within major corporations and within academia. For
example, today most major corporate websites lay emphasis on commitment to
promoting non-economic social values under a variety of headings (e.g. ethics codes,
social responsibility charters). In some cases, corporations have re-branded their core
values in the light of business ethical considerations (e.g. BP's "beyond petroleum"
environmental tilt).

The term CSR came in to common use in the early 1970s, after many multinational
corporations formed, although it was seldom abbreviated. The term stakeholder,
meaning those impacted by an organization's activities, was used to describe corporate
owners beyond shareholders as a result of an influential book by R Freeman in 1984.

Whilst there is no recognized standard for CSR, public sector organizations (the United
Nations for example) adhere to the Triple Bottom Line (TBL). It is widely accepted that
CSR adheres to similar principles but with no formal act of legislation. The UN has
developed the Principles for Responsible Investment as guidelines for investing entities

The scale and nature of the benefits of CSR for an organization can vary depending on
the nature of the enterprise, and are difficult to quantify, though there is a large body of
literature exhorting business to adopt measures beyond financial ones. However,
businesses may not be looking at short-run financial returns when developing their CSR
strategy.

The definition of CSR used within an organization can vary from the strict "stakeholder
impacts" definition used by many CSR advocates and will often include charitable
efforts and volunteering. CSR may be based within the human resources, business
development or public relations departments of an organisation, or may be given a
separate unit reporting to the CEO or in some cases directly to the board. Some
companies may implement CSR-type values without a clearly defined team or
programme.

Abhishek kumar yadav 520844307 Page 29


INTERNATIONAL BUSINESS MANAGEMENT

(MB-0037)

ASSIGNMENT-1

Q.1 Write a short note on ‘Globalization’.

Ans. The term "globalization" has acquired considerable emotive force. Some view it as a
process that is beneficial – a key to future world economic development – and also
inevitable and irreversible. Others regard it with hostility, even fear, believing that it
increases inequality within and between nations, threatens employment and living
standards and thwarts social progress. This brief offers an overview of some aspects of
globalization and aims to identify ways in which countries can tap the gains of this
process, while remaining realistic about its potential and its risks.

Globalization offers extensive opportunities for truly worldwide development but it is not
progressing evenly. Some countries are becoming integrated into the global economy
more quickly than others. Countries that have been able to integrate are seeing faster
growth and reduced poverty.

Economic "globalization" is a historical process, the result of human innovation and


technological progress. It refers to the increasing integration of economies around the
world, particularly through trade and financial flows. The term sometimes also refers to
the movement of people (labor) and knowledge (technology) across international
borders. There are also broader cultural, political and environmental dimensions of
globalization that are not covered here.

At its most basic, there is nothing mysterious about globalization. The term has come
into common usage since the 1980s, reflecting technological advances that have made
it easier and quicker to complete international transactions – both trade and financial

Abhishek kumar yadav 520844307 Page 30


flows. It refers to an extension beyond national borders of the same market forces that
have operated for centuries at all levels of human economic activity – village markets,
urban industries, or financial centers.

Globalization is not just a recent phenomenon. Some analysts have argued that the
world economy was just as globalized 100 years ago as it is today. But today commerce
and financial services are far more developed and deeply integrated than they were at
that time. The most striking aspect of this has been the integration of financial markets
made possible by modern electronic communication.

There are four aspects of globalization:

1. Trade: Developing countries as a whole have increased their share of world trade –
from 19 percent in 1971 to 29 percent in 1999. For instance, the newly industrialized
economies (NIEs) of Asia have done well, while Africa as a whole has fared poorly.
The composition of what countries export is also important. The strongest rise by far
has been in the export of manufactured goods. The share of primary commodities in
world exports – such as food and raw materials – that are often produced by the
poorest countries, has declined.

2. Capital movements: Globalization sharply increased private capital flows to


developing countries during much of the 1990s. It also shows that:

• the increase followed a particularly "dry" period in the 1980s;

• net official flows of "aid" or development assistance have fallen significantly


since the early 1980s; and

• the composition of private flows has changed dramatically. Direct foreign


investment has become the most important category. Both portfolio investment
and bank credit rose but they have been more volatile, falling sharply in the
wake of the financial crises of the late 1990s.

3. Movement of people: Workers move from one country to another partly to find
better employment opportunities. The numbers involved are still quite small, but in
the period 1965-90, the proportion of labor forces round the world that was foreign
born increased by about one-half. Most migration occurs between developing
countries. But the flow of migrants to advanced economies is likely to provide a
means through which global wages converge. There is also the potential for skills to
be transferred back to the developing countries and for wages in those countries to
rise.

Abhishek kumar yadav 520844307 Page 31


4. Spread of knowledge (and technology): Information exchange is an integral, often
overlooked, aspect of globalization. For instance, direct foreign investment brings not
only an expansion of the physical capital stock, but also technical innovation. More
generally, knowledge about production methods, management techniques, export
markets and economic policies is available at very low cost, and it represents a
highly valuable resource for the developing countries.

Q.2 Describe the positives of trade liberalization.

Ans. Policies that make an economy open to trade and investment with the rest of the world
are needed for sustained economic growth. The evidence on this is clear. No country in
recent decades has achieved economic success, in terms of substantial increases in
living standards for its people, without being open to the rest of the world. In contrast,
trade opening (along with opening to foreign direct investment) has been an important
element in the economic success of East Asia.

Opening up their economies to the global economy has been essential in enabling
many developing countries to develop competitive advantages in the manufacture of
certain products. In these countries, defined by the World Bank as the "new
globalizers," the number of people in absolute poverty declined by over 120 million (14
percent) between 1993 and 1998.

There is considerable evidence that more outward-oriented countries tend consistently


to grow faster than ones that are inward-looking. Indeed, one finding is that the benefits
of trade liberalization can exceed the costs by more than a factor of 10. Countries that
have opened their economies in recent years, including India, Vietnam, and Uganda,
have experienced faster growth and more poverty reduction. On average, those
developing countries that lowered tariffs sharply in the 1980s grew more quickly in the
1990s than those that did not.

Freeing trade frequently benefits the poor especially. Developing countries can ill-afford
the large implicit subsidies, often channeled to narrow privileged interests that trade
protection provides. Moreover, the increased growth that results from free trade itself
tends to increase the incomes of the poor in roughly the same proportion as those of
the population as a whole. New jobs are created for unskilled workers, raising them into
the middle class. Overall, inequality among countries has been on the decline since
1990, reflecting more rapid economic growth in developing countries, in part the result
of trade liberalization.

Although there are benefits from improved access to other countries’ markets, countries
benefit most from liberalizing their own markets. The main benefits for industrial
countries would come from the liberalization of their agricultural markets. Developing

Abhishek kumar yadav 520844307 Page 32


countries would gain about equally from liberalization of manufacturing and agriculture.
The group of low-income countries, however, would gain most from agricultural
liberalization in industrial countries because of the greater relative importance of
agriculture in their economies.

Further liberalization – by both industrial and developing countries – will be needed to


realize trade’s potential as a driving force for economic growth and development.
Greater efforts by industrial countries and the international community more broadly, are
called for to remove the trade barriers facing developing countries, particularly the
poorest countries. Although quotas under the so-called Multi-fibre Agreement are due to
be phased out by 2005, speedier liberalization of textiles and clothing and of agriculture
is particularly important. Similarly, the elimination of tariff peaks and escalation in
agriculture and manufacturing also needs to be pursued. In turn, developing countries
would strengthen their own economies (and their trading partners’) if they made a
sustained effort to reduce their own trade barriers further.

Enhanced market access for the poorest developing countries would provide them with
the means to harness trade for development and poverty reduction. Offering the poorest
countries duty – and quota – free access to world markets would greatly benefit these
countries at little cost to the rest of the world. The recent market-opening initiatives of
the EU and some other countries are important steps in this regard. To be completely
effective, such access should be made permanent, extended to all goods, and
accompanied by simple, transparent rules of origin. This would give the poorest
countries the confidence to persist with difficult domestic reforms and ensure effective
use of debt relief and aid flows.

Q.3 Write a short note on GATT and WTO, highlighting the difference between the
two.

Ans. General Agreement on Tariff and Trade( GATT):

The GATT, was established on a provisional basis after the Second World War in the
wake of other new multilateral institutions dedicated to international economic
cooperation – notably the "Britton Woods" institutions now known as the World Bank
and the International Monetary Fund.

The original 23 GATT countries were among over 50 which agreed a draft Charter for
an International Trade Organization (ITO) – a new specialized agency of the United
Nations. The Charter was intended to provide not only world trade disciplines but also
contained rules relating to employment, commodity agreements, restrictive business
practices, international investment and services.

In an effort to give an early boost to trade liberalization after the Second World War and

Abhishek kumar yadav 520844307 Page 33


to begin to correct the large overhang of protectionist measures which remained in
place from the early 1930s-tariff negotiations were opened among the 23 founding
GATT "contracting parties" in 1946. This first round of negotiations resulted in 45,000
tariff concessions affecting $10 billion or about one-fifth – of world trade. It was also
agreed that the value of these concessions should be protected by early and largely
"provisional" acceptance of some of the trade rules in the draft ITO Charter. The tariff
concessions and rules together became known as the General Agreement on Tariffs
and Trade and entered into force in January 1948.

Although the ITO Charter was finally agreed at a UN Conference on Trade and
Employment in Havana in March 1948, ratification in national legislatures proved
impossible in some cases. When the United States’ government announced, in 1950,
that it would not seek Congressional ratification of the Havana Charter, the ITO was
effectively dead. Despite its provisional nature, the GATT remained the only multilateral
instrument governing international trade from 1948 until the establishment of the WTO.

Although, in its 47 years, the basic legal text of the GATT remained much as it was in
1948, there were additions in the form of "plural-lateral” voluntary membership
agreements and continual efforts to reduce tariffs. Much of this was achieved through a
series of "trade rounds".

The biggest leaps forward in international trade liberalization have come through
multilateral trade negotiations, or "trade rounds", under the auspices of GATT – the
Uruguay Round was the latest and most extensive.

The limited achievement of the Tokyo Round, outside the tariff reduction results, was a
sign of difficult times to come. GATT’s success in reducing tariffs to such a low level,
combined with a series of economic recessions in the 1970s and early 1980s, drove
governments to devise other forms of protection for sectors facing increased overseas
competition. High rates of unemployment and constant factory closures led
governments in Europe and North America to seek bilateral market-sharing
arrangements with competitors and to embark on a subsidies race to maintain their
holds on agricultural trade. Both these changes undermined the credibility and
effectiveness of GATT.

WTO

World Trade Organization came into existence in 1995 after the desolation of General
Agreement on Tariff and Trade (GATT).

The WTO’s overriding objective is to help trade flow smoothly, freely, fairly and
predictably. It does this by:

Abhishek kumar yadav 520844307 Page 34


 Administering trade agreements
 Acting as a forum for trade negotiations
 Settling trade disputes
 Reviewing national trade policies
 Assisting developing countries in trade policy issues, through technical
assistance and training programs
 Cooperating with other international organizations
The WTO has nearly 150 members, accounting for over 97% of world trade. Around 30
others are negotiating membership. Decisions are made by the entire membership. This
is typically by consensus. A majority vote is also possible but it has never been used in
the WTO, and was extremely rare under the WTO’s predecessor, GATT. The WTO’s
agreements have been ratified in all members’ parliaments.

The WTO’s top level decision-making body is the Ministerial Conference which meets
at least once every two years. Below this is the General Council which meets several
times a year in the Geneva headquarters. The General Council also meets as the Trade
Policy Review Body and the Dispute Settlement Body. At the next level, the Goods
Council, Services Council and Intellectual Property (TRIPS) Council report to the
General Council.

Numerous specialized committees, working groups and working parties deal with
the individual agreements and other areas such as the environment, development,
membership applications and regional trade agreements.

The WTO Secretariat, based in Geneva, has around 600 staff and is headed by a
director-general. Its annual budget is roughly 160 million Swiss francs. It does not have
branch offices outside Geneva. Since decisions are taken by the members themselves,
the Secretariat does not have the decision-making role that other international
bureaucracies are given with.

The WTO is run by its member governments. All major decisions are made by the
membership as a whole, either by ministers (who meet at least once every two years) or
by their ambassadors or delegates (who meet regularly in Geneva). Decisions are
normally taken by consensus.

Difference between WTO and GATT:-

The World Trade Organization is not a simple extension of GATT; on the contrary, it
completely replaces its predecessor and has a very different character. Among the

Abhishek kumar yadav 520844307 Page 35


principal differences are the following:

a. The GATT was a set of rules, a multilateral agreement, with no institutional


foundation, only a small associated secretariat which had its origins in the attempt
to establish an International Trade Organization in the 1940s. The WTO is a
permanent institution with its own secretariat.

b. The GATT was applied on a "provisional basis" even if, after more than forty years,
governments chose to treat it as a permanent commitment. The WTO
commitments are full and permanent.

c. The GATT rules applied to trade in merchandise goods. In addition to goods, the
WTO covers trade in services and trade-related aspects of intellectual property.

d. While GATT was a multilateral instrument, by the 1980s many new agreements
had been added of a plural-lateral, and therefore selective, nature. The agreements
which constitute the WTO are almost all multilateral and, thus, involve
commitments for the entire membership.

e. The WTO dispute settlement system is faster, more automatic, and thus much less
susceptible to blockages, than the old GATT system. The implementation of WTO
dispute findings will also be more easily assured.

Q.4 Think of any MNC and analyze its business strategy orientation.

Ans. Multinational companies (MNC) may pursue business strategies that are home country
– oriented or host country – oriented or world – oriented.

Perlmutter uses such terms as ethnocentric, polycentric and geocentric. However,


"ethnocentric" is misleading because it focuses on race or ethnicity, especially when the
home country itself is populated by many different races, whereas "polycentric" loses its
meaning when the MNCs operate only in one or two foreign countries.

According to Franklin Root (1994), an MNC is a parent company that

a. engages in foreign production through its affiliates located in several countries,


b. exercises direct control over the policies of its affiliates,
c. implements business strategies in production, marketing, finance and staffing that
transcend national boundaries.
Business strategy of a MNC can be analyzed with the help of Three Stages of Evolution

1. Export stage

• initial inquiries - firms rely on export agents

Abhishek kumar yadav 520844307 Page 36


• expansion of export sales
• further expansion - foreign sales branch or assembly operations (to save
transport cost)
2. Foreign Production Stage

There is a limit to foreign sales (tariffs, NTBs).Once the firm chooses foreign
production as a method of delivering goods to foreign markets, it must decide
whether to establish a foreign production subsidiary or license the technology to a
foreign firm.

Licensing is usually first experience (because it is easy)


• it does not require any capital expenditure
• it is not risky
• payment = a fixed % of sales
e.g.: Kentucky Fried Chicken in the U.K.
Problem that may arise while following a particular business strategy: The
mother firm may find it difficult in exercise of any managerial control over the
licensee (as it is independent).

Secondly, the licensee may transfer industrial secrets to another independent firm,
thereby creating a rival.

The next stage for supplementing any particular business strategy is


Investments involved.

It requires the decision of top management because it is a critical step.

 it is risky (lack of information) (for example-US firms tend to establish


subsidiaries in Canada first. Singer Manufacturing Company established its
foreign plants in Scotland and Australia in the 1850s)
 plants are established in several countries
 licensing is switched from independent producers to its subsidiaries.
 export continues
3. Multinational Stage: The company becomes a multinational enterprise when it
begins to plan, organize and coordinate production, marketing, R&D, financing, and
staffing. For each of these operations, the firm must find the best location.

This is how a MNC decides its business strategy orientation.

Q.5 What does FDI stand for? Why do MNCs opt for FDI to enter international market?

Abhishek kumar yadav 520844307 Page 37


Ans. FDI stands for Foreign Direct Investment. New MNCs do not pop up randomly in foreign
nations. It is the result of conscious planning by corporate managers. Investment flows
from regions of low anticipated profits to those of high returns. When MNC
incorporated in one country, invests in another country, it is said that the FDI has flowed
into the other country from some foreign origin.

The main reasons for MNCs to opt for FDI to enter international market is stated as
follows:

1. Growth motive: A company may have reached a plateau satisfying domestic


demand, which is not growing. Looking for new markets.

2. Protection in the importing countries : Foreign direct investment is one way to


expand. FDI is a means to bypassing protective instruments in the importing country.
European Community imposed common external tariff against outsiders. US
companies circumvented these barriers by setting up subsidiaries. Japanese
corporations located auto assembly plants in the US, to bypass VERs.

3. High Transportation Costs : Transportation costs are like tariffs in that they are
barriers which raise consumer prices. When transportation costs are high,
multinational firms want to build production plants close to the market in order to
save transportation costs. Multinational firms that invested and built production
plants in the United States are better off than the exporting firms that utilized New
Orleans port to ship and distribute products through New Orleans, provided that they
built plants in a safe area.

4. Exchange Rate Fluctuations: Japanese firms invest here to produce heavy


construction machines to avoid excessive exchange rate fluctuations. Also,
Japanese automobile firms have plants to produce automobile parts. For instance,
Toyota imports engines and transmissions from Japanese plants, and produce the
rest in the U.S.

5. Market competition: The most certain method of preventing actual or potential


competition is to acquire foreign businesses. GM purchased Monarch (GM Canada)
and Opel (GM Germany). It did not buy Toyota, Datsun (Nissan) and Volkswagen.
They later became competitors.

6. Cost reduction: United Fruit has established banana-producing facilities in Honduras.


Cheap foreign labour. Labour costs tend to differ among nations. MNCs can hold
down costs by locating part of all their productive facilities abroad. (Maquildoras)

Abhishek kumar yadav 520844307 Page 38


Q.6 Viewing culture as a multi-level construct, describe various levels it consists of.

Ans. There are two kinds of approach construct of culture. One is a multi-level approach,
viewing culture as a multi-level construct that consists of various levels nested within
each other from the most macro-level of a global culture, through national cultures,
organizational cultures, group cultures, and cultural values that are represented in the
self at the individual level.

The second is based on Schein’s (1992) model viewing culture as a multi – layer
construct consisting of the most external layer of observed artifacts and behaviours,
the deeper level of values, which is testable by social consensus, and the deepest level
of basic assumption, which is invisible and taken for granted.

The present model proposes that culture as a multi – layer construct exists at all levels
– from the global to the individual – and that at each level change first occurs at the
most external layer of behaviour, and then, when shared by individuals who belong to
the same cultural context, it becomes a shared value that characterizes the aggregated
unit (group, organizations, or nations).

In the model, the most macro-level is that of a global culture being created by global
networks and global institutions that cross national and cultural borders.

Figure-1: The dynamic of top-down–bottom-up processes across levels of culture.

Given the dominance of Western MNCs, the values that dominate the global context are
often based on a free market economy, democracy, acceptance and tolerance of
diversity, respect of freedom of choice, individual rights, and openness to change.

Below the global level are nested organizations and networks at the national level with
their local cultures varying from one nation or network to another. Further down are

Abhishek kumar yadav 520844307 Page 39


local organizations, and although all of them share some common values of their
national culture, they vary in their local organizational cultures, which are also shaped
by the type of industry that they represent, the type of ownership, the values of the
founders, etc. Within each organization are sub-units and groups that share the
common national and organizational culture, but that differ from each other in their unit
culture on the basis of the differences in their functions (e.g., R&D vs manufacturing),
their leaders’ values, and the professional and educational level of their members. At
the bottom of this structure are individuals who through the process of socialization
acquire the cultural values transmitted to them from higher levels of culture. Individuals
who belong to the same group share the same values that differentiate them from other
groups and create a group – level culture through a bottom-up process of aggregation
of shared values. For example, employees of an R&D unit are selected into the unit
because of their creative cognitive style and professional expertise. Their leader also
typically facilitates the display of these personal characteristics because they are crucial
for developing innovative products. Thus, all members of this unit share similar core
values, which differentiate them from other organizational units. Groups that share
similar values create the organizational culture through a process of aggregation, and
local organizations that share similar values create the national culture that is different
from other national cultures.

Both top-down and bottom-up processes reflect the dynamic nature of culture, and
explain how culture at different levels is being shaped and reshaped by changes that
occur at other levels, either above it through top-down processes or below it through
bottom-up processes. Similarly, changes at each level affect lower levels through a top-
down process, and upper levels through a bottom-up process of aggregation.

Global organizations and networks are being formed by having local-level organizations
join the global arena. That means that there is a continuous reciprocal process of
shaping and reshaping organizations at both levels. For example, multinational
companies that operate in the global market develop common rules and cultural values
that enable them to create a synergy between the various regions, and different parts of
the multinational company. These global rules and values filter down to the local
organizations that constitute the global company, and, over time, they shape the local
organizations. Reciprocally, having local organizations join a global company may
introduce changes into the global company because of its need to function effectively
across different cultural boarders.

Abhishek kumar yadav 520844307 Page 40


INTERNATIONAL BUSINESS MANAGEMENT

(MB-0037)

ASSIGNMENT-2

Q.1 Write a short note on Bill of Lading.

Ans. A Bill of Lading is a type of document that is used to acknowledge the receipt of a
shipment of goods and is an essential document in transporting goods overland to the
exporter’s international carrier. A through Bill of Lading involves the use of at least two
different modes of transport from road, rail, air and sea. The term derives from the noun
"bill", a schedule of costs for services supplied or to be supplied, and from the verb "to
lade" which means to load a cargo onto a ship or other form of transport.

In addition to acknowledging the receipt of goods, a Bill of Lading indicates the


particular vessel on which the goods have been placed, their intended destination, and
the terms for transporting the shipment to its final destination. Inland, ocean, through,
and airway bill are the names given to bills of lading.

Q.2 In terms of product design, most products fall in between the spectrum of two
extremes. What are these two extremes? Describe on these extremes in brief.

Ans. Most products decisions lie between two extremes. One of the extreme is to sell
globally standardized products and the other extreme is to sell adapted products.

Changes in design are largely dictated by whether they would improve the prospects of
greater sales, and this, over the accompanying costs. Changes in design are also
subject to cultural pressures. The more culture-bound the product is, for example food,
the more adaptation is necessary. Most products fall in between the spectrum of
"standardization" to "adaptation" extremes. The application the product is put to also
affect the design. In the UK, railway engines were designed from the outset to be
sophisticated because of the degree of competition, but in the US this was not the case.

Abhishek kumar yadav 520844307 Page 41


In order to burn the abundant wood and move the prairie debris, large smoke stacks
and cowcatchers were necessary. In agricultural implements a mechanized cultivator
may be a convenience item in a UK garden, but in India and Africa it may be essential
equipment. As stated earlier "perceptions" of the product’s benefits may also dictate the
design. A refrigerator in Africa is a very necessary and functional item, kept in the
kitchen or the bar. In Mexico, the same item is a status symbol and, therefore, kept in
the living room.

Factors encouraging standardization are:


a. economies of scale in production and marketing
b. consumer mobility – the more consumers travel the more is the demand
c. technology
d. image, for example "Japanese", "made in".
The latter can be a factor both to aid or to hinder global marketing development.
Nagashima1 (1977) found the "made in USA" image has lost ground to the "made in
Japan" image. In some cases "foreign made" gives advantage over domestic products.
In Zimbabwe one sees many advertisements for "imported", which gives the product,
advertised a perceived advantage over domestic products. Often a price premium is
charged to reinforce the "imported means quality" image. If the foreign source is
negative in effect, attempts are made to disguise or hide the fact through, say,
packaging or labelling. Mexicans are loathing taking products from Brazil. By putting a
"made in elsewhere" label on the product this can be overcome, provided the products
are manufactured elsewhere even though its company maybe Brazilian.

Factors encouraging adaptation are:

a. Differing usage conditions – These may be due to climate, skills, level of literacy,
culture or physical conditions. Maize, for example, would never sell in Europe rolled
and milled as in Africa. It is only eaten whole, on or off the cob. In Zimbabwe,
kapenta fish can be used as a relish, but wilt always be eaten as a "starter" to a
meal in the developed countries.

b. General market factors – incomes, tastes etc. Canned asparagus may be very
affordable in the developed world, but may not sell well in the developing world.

c. Government – taxation, import quotas, non tariff barriers, labelling, health


requirements. Non tariff barriers are an attempt, despite their supposed impartiality,
at restricting or eliminating competition. A good example of this is the Florida
tomato growers, cited earlier, who successfully got the US Department of
Agriculture to issue regulations establishing a minimum size of tomatoes marketed
in the United States. The effect of this was to eliminate the Mexican tomato

Abhishek kumar yadav 520844307 Page 42


industry which grew a tomato that fell under the minimum size specified. Some
non-tariff barriers may be legitimate attempts to protect the consumer, for example
the ever stricter restrictions on horticultural produce insecticides and pesticides use
may cause African growers a headache, but they are deemed to be for the public
good.

d. History. Sometimes, as a result of colonialism, production facilities have been


established overseas. Eastern and Southern Africa is littered with examples. In
Kenya, the tea industry is a colonial legacy, as are the sugar industry of Zimbabwe
and the coffee industry of Malawi. These facilities have long been adapted to local
conditions.

e. Financial considerations. In order to maximize sales or profits the organization may


have no choice but to adapt its products to local conditions.

f. Pressure. Sometimes, as in the case of the EU, suppliers are forced to adapt to the
rules and regulations imposed on them if they wish to enter into the market.

Q.3 A Europe based MNC wants to introduce its fruit juice drink in India. What
product strategy of international marketing do you think will be suitable for its
product?

Ans. There are five major product strategies in international marketing.

a. Product communications extension-This strategy is very low cost and merely


takes the same product and communication strategy into other markets. However, it
can be risky if mis-judgements are made. For example, CPC International believed
the US consumer would take to dry soups, which dominate the European market. It
did not work.

b. Extended product – communications adaptation-If the product basically fits the


different needs or segments of a market it may need an adjustment in marketing
communications only. Again this is a low cost strategy, but different product
functions have to be identified and a suitable communications mix developed.

c. Product adaptation – communications extension: The product is adapted to fit


usage conditions but the communication stays the same. The assumption is that the
product will serve the same function in foreign markets under different usage
conditions.

d. Product adaptation–communications adaptation-: Both product and


communication strategies need attention to fit the peculiar need of the market.

Abhishek kumar yadav 520844307 Page 43


e. Product invention-This needs a totally new idea to fit the exclusive conditions of the
market. This is very much a strategy which could be ideal in a Third World situation.
The development costs may be high, but the advantages are also very high.

The choice of strategy will depend on the most appropriate product/market analysis and
is a function of the product itself defined in terms of the function or need it serves, the
market defined in terms of the conditions under which the product is used, the
preferences of the potential customers and the ability to buy the product in question,
and the costs of adaptation and manufacture to the company considering these product
– communications approaches.

In this case the Euopean MNC should undertake the Product adaptation –
communications adaptation strategy. This is because the fruit drink is entering a
totally a new market, India, whose preferences are quite dissimilar from Europe. At the
same time as culturally India is different from Europe there is need for adaptation in
communications also as the product will get exposed to a whole new market segment.

Q.4 Describe ‘hard data’ vis-à-vis ‘soft data’.

Ans. "Hard" data refers to relatively quantifiable measures such as a country’s GDP, number
of telephones per thousand residents, and birth rates (although even these supposedly
"objective" factors may be subject to some controversy due to differing definitions and
measurement approaches across countries).

In contrast, "soft" data refers to more subjective issues such as country history or
culture. It should be noted that while the "hard" data is often more convenient and
seemingly objective, the "soft" data is frequently as important, if not more so, in
understanding a market.

Q.5 What is ‘Protectionism’? How can Protectionism be justified?

Ans. Protectionism is levying of additional tariffs or other protectionist measures by other


countries in retaliation, reduced competition (which results in inflation and less choice
for consumers), a weakening of the trade balance (due in part to diminished export
abilities resulting from foreign retaliations and in part because of the domestic currency
loses power as there is less demand for it). This may lead to a vicious cycle of trade
wars as each country responds to the other with a "tit for tat."

Although trade generally benefits a country as a whole, powerful interests within


countries frequently put obstacles – i.e., they seek to inhibit free trade.

There are several ‘protectionism’ can be done:

a. Tariff barriers: A duty, or tax or fee, is put on products imported. This is usually a

Abhishek kumar yadav 520844307 Page 44


percentage of the cost of the good.

b. Quotas: A country can export only a certain number of goods to the importing
country. For example, Mexico can export only a certain quantity of tomatoes to the
United States, and Asian countries can send only a certain quota of textiles here.

c. "Voluntary" export restraints: These are not official quotas, but involve
agreements made by countries to limit the amount of goods they export to an
importing country. Such restraints are typically motivated by the desire to avoid
more stringent restrictions if the exporters do not agree to limit themselves. For
example, Japanese car manufacturers have agreed to limit the number of
automobiles they export to the United States.

d. Subsidies to domestic products: If the government supports domestic producers


of a product, these may end up with a cost advantage relative to foreign producers
who do not get this subsidy. U.S. honey manufacturers receive such subsidies.

e. Non-tariff barriers, such as differential standards in testing foreign and domestic


products for safety, disclosure of less information to foreign manufacturers needed
to get products approved, slow processing of imports at ports of entry, or arbitrary
laws which favour domestic manufacturers.

Justifications for protectionism:

Several justifications have been made for the practice of protectionism. Some appear to
hold more merit than others:

a. Protection of an "infant" industry: Costs are often higher, and quality lower, when
an industry first gets started in a country, and it thus be very difficult for that country
to compete. However, as the industry in the country matures, it may be better able
to compute. Thus, for example, some countries have attempted to protect their
domestic computer markets while they gained strength. The U.S. attempted to
protect its market for small autos. American manufacturers were caught
unprepared for the switch in demand away from the larger cars. This is generally
an accepted reason in trade agreements, but the duration of this protection must
be limited (e.g., a maximum of five to ten years).

b. Resistance to unfair foreign competition: The U.S. sugar industry contends that
most foreign manufacturers subsidize their sugar production, so the U.S. must
follow to remain competitive. This argument will hold little merit with the dispute
resolution mechanism available through the World Trade Organization.

c. Preservation of a vital domestic industry: The U.S. wants to be able to produce its

Abhishek kumar yadav 520844307 Page 45


own defence products, even if foreign imports would be cheaper, since the U.S.
does not want to be dependent on foreign manufacturers with whose countries
conflicts may arise. Similarly, Japan would prefer to be able to produce its own
food supply despite its exorbitant costs. For an industry essential to national
security, this may be a compelling argument, but it is often used for less compelling
ones (e.g., manufactures of funeral caskets or honey).

d. Intervention into a temporary trade balance: A country may want to try to reverse a
temporary decline in trade balances by limiting imports. In practice, this does not
work since such moves are typically met by retaliation.

e. Maintenance of domestic living standards and preservation of jobs. Import


restrictions can temporarily protect domestic jobs, and can in the long run protect
specific jobs (e.g., those of auto makers, farmers, or steel workers). This is less of
an accepted argument – these workers should instead by retrain to work in jobs
where their country has a relative advantage.

f. Retaliation: The proper way to address trade disputes is now through the World
Trade Organization. In the past, where enforcement was less available, this might
have been a reasonable argument.

Note that while protectionism generally hurts a country overall, it may be beneficial to
specific industries or other interest groups. Thus, while sugar price supports are bad for
consumers in general, producers are an organized group that can exert a great deal of
influence. In contrast, the individual consumer does not have much of an incentive to
take action to save.

Q.6 Describe various entry strategies available to a firm when it wants to enter a
foreign market.

Ans. The various strategies available to a firm when it enters a foreign market are as
follows:-

1. Supplying Products to Foreign Buyers : Foreign production is not always an


answer. Foreign markets can be better served by exporting, rather than by creating
a foreign subsidiary if there are economies of scale. If large scale production
reduces unit cost, it is better to concentrate production in one place. MES is the
minimum rate of output at which Average Cost (AC) is minimized. If minimum
efficient scale (MES) is not achieved, then export.

In other words, if there is excess capacity, why not utilize that and export outputs to
other countries? There is no point in creating another plant overseas when domestic
capacity is not fully utilized. If the foreign demand exceeds the minimum efficient

Abhishek kumar yadav 520844307 Page 46


scale, then FDI.

Figure -2 : Minimum efficient scale and FDI.

2. International Joint Ventures: JV is a business organization established by two or


more companies that combines their skills and assets.

 A JV is formed by two businesses that conduct business in a third country. (US


firm + British firm jointly operate in the Middle East)

 Joint venture with a local firm (GM + Shanghai Automobile Company)

 Joint venture may include local government (Bechtel Company-US;


Messerschmitt – Boelkow – Blom, Germany; Iran Oil Investment Company;
National Iranian Oil Company)

International JV has certain benefits. These are

 Large capital costs – costs are too large for a single company

 Protection – LDC governments close their borders to foreign companies

 Bypass protectionism. e.g.: US workers assemble Japanese parts. The finished


goods are sold to the US consumers.

 The new venture increases production, lowers price to consumers.

 The new business is able to enter the market that neither parent could have
entered singly.

 Cost reductions (otherwise, no joint ventures will be formed)

Abhishek kumar yadav 520844307 Page 47


 increased market power

3. Tax Policy towards MNCs: Operating in many countries, MNCs are subject to
multiple tax jurisdictions, i.e., they must pay taxes to several countries. National tax
systems are exceedingly complex and differ between countries. Differences among
national income tax systems affect the decisions of managers of MNCs, regarding
the location of subsidiaries, financing, and the transfer prices (the prices of products
and assets transferred between various units of MNCs).

Multiple Tax Jurisdictions creates two problems, overlapping and under lapping
jurisdictions. When overlapping occurs, two or more governments claim tax
jurisdictions over the same income of an MNC. The overlapping may result in double
taxation.

Conversely, when under lapping occurs, an MNC falls between tax jurisdictions and
escape taxation. Under lapping encourages tax avoidance. National governments
may choose a territorial jurisdiction or national tax jurisdiction or both.

4. Transfer Pricing : MNCs try to reduce their overall tax burden. An MNC reports
most of its profits in a low – tax country, even though the actual profits are earned in
a high tax country.

tp = tax rate in the parent country ; th = tax rate in the host country

If tp > th, then under price its exports to the subsidiary in the host country, and
overprice its imports from the subsidiary in order to lower tax.

Purpose is to manipulate prices between headquarter and the subsidiaries so that


profits are highest in the low tax country.

Thus, a multinational company’s overall tax could be paid at the minimum of all tax
rates of the countries in which it operates.

5. Taxation and Gains from Factor Mobility : It is seen that US firms invest overseas
because the returns are higher there. Assume both countries have the same
corporate tax rates = 40%

US Canada

Pretax profits 10% 12%

Tax 4% 4.8%

Net to investors 6% 7.2%

Abhishek kumar yadav 520844307 Page 48


Total Gains from domestic investment = 10% (= 4% + 6%) because tax revenues
can be used for public purposes.

Total Gains from foreign investment = 7.2% (because US government gets nothing).
The tax revenue which could have been used to build US highways would be used
by Canadian government to build their highways.

A firm has to evaluate all such kinds of complex factors to fix up any strategy before
choosing to enter a foreign market

Abhishek kumar yadav 520844307 Page 49


SERVICE MARKETING AND CUSTOMER RELATIONSHIP
MANAGEMENT

(MK-0006)

ASSIGNMENT-1

Q. 1 Explain the bases of classification of services.

Ans. A service is an activity which has some element of intangibility associated with it, which
involves some interaction with customers or with property in their possession, and does
not result in a transfer of ownership. A change in condition may occur and production of
the service may or may not be closely associated with a physical product. Goods are
simply the objects and services are simply the provision of such information of goods by
the seller and the buyers. The goods are visible and shall be purchased by the buyers
and sold by the sellers, whereas the services are the ones received by the guests and
provided by the goods sellers.

The objective in any service classification scheme is to get a deeper understanding of


the service product. Despite the diversity in the range of service products, it is possible
to classify and explore them on the basis of certain factors. There have been more than
sixteen studies regarding the classification schemes. Naturally, some are worthwhile in
developing marketing strategies, with others suffer shortcomings.

According to Christopher Lovelock following issues should be considered for the


classification of services:

a. Service industries continue to be dominated by operations, with the service


managers insisting that their tasks and challenges in their industry are unique and
have nothing in common with those from other service industries.

b. A managerial mind set evident in many service argues that the marketing of a
service industry has nothing in common with the marketing of another service
industry. For example, the marketing of an airline service has nothing in common
with marketing of a banking service or a financial service.

c. Classification schemes should offer strategic marketing insights so as to have


managerial value. Any other simple classification would be insufficient.

As per Payne following factors determine classification of services:

a. Type of services,

Abhishek kumar yadav 520844307 Page 50


b. Type of seller,

c. Type of purchaser,

d. Demand characteristics,

e. Rented versus owned services,

f. Degree of intangibility,

g. Buying motives,

h. Equipment based versus people based,

i. Amount of customer contact,

j. Service delivery requirements,

k. Degree of customization and

l. Degree of labour intensity.

According to Kotler and Station, services classification must be equipment based or


people based. The service may meet a personal need or a business need, according to
the difference in their objectives and the ownership. The objective may either be for
profit or non-profit and the ownership may be private or public. Accordingly services
may be classified as below:

a. Housing,

b. Household operations like utilities, house repairs, repairs of equipments in the


house etc.,

c. Recreation,

d. Personal care like laundry, beauty care etc.,

e. Medical and other health care,

f. Private education,

g. Business and other professional services,

h. Insurance and financial,

i. Transportation and

j. Communication.

Lovelock’s Classification Schemes is based on following factors:

a. Nature of the Service Act : In this scheme, the nature of the service act, i.e
intangibility etc., and the recipients of the services are considered. It has wide

Abhishek kumar yadav 520844307 Page 51


dimensions and considers how the customer’s presence, physically or mentally,
is required during service delivery and the benefits gained by the customer. It
also considers how the customers change on receipt of the service. It leads to the
consideration of location and scheduling convenience for the presence of the
customer to utilize the service.

b. Relationship with Customer: This classification matrix contrasts the nature of


service delivery and ascertains whether there is a formal relationship between the
customer and service marketer. The knowledge of customers’ identities and
addresses gained to meet out specific offers to specify customers by targeted
direct marketing makes market segmentation easier and provides insights into
trade off between pricing and usage rates. In the absence of personal
relationship, services may be provided by continuous delivery or by discrete
transaction.

c. Customization and Judgment in Service Delivery : In this classification, the


degree of customization of service characteristics is contrasted with the degree of
judgment required by customer. The customized information is very important to
the service marketers. They have to balance the cost of custom made service
with a standard service and so the service marketers often seek to limit the
number of options while deciding the extent of customization.

d. Nature of Demand and Supply for Services: This classification contrasts the
nature of demand fluctuations over time and the extent to which the supply is
constrained. The business may be lost to another service provider if the demand
exceeds the supply of any particular service. This classification is useful to
contrast different supply demand situations which affect all service marketers. It
emphasizes in establishing demand patterns overtime, and to understand their
existence. Then development of strategies is considered to help change pattern
of demand so as to make them more favourable to the service provider.

e. Method of Service Delivery: The fifth classification focuses attention on


examination of the availability of service outlets, ranging from single to multiple
sites, and the nature of the interaction between the customer and the service
provider. Distribution issues relating to the method of delivery are focused upon
customer convenience which is of important consideration here.

These classification systems of Lovelock provide frame work for service marketers to
consider both the nature of their business and to what extent they share common
characteristics with other seemingly unrelated service businesses.

Abhishek kumar yadav 520844307 Page 52


Other classification:

Thomas classified the services on the basis of type of equipment or people rendering
the services

a. Equipment based : In this case, the equipment’s or the machines being utilized
for service position are important while people play a secondary role. Some of the
examples are automatic vending machines, automatic car washes and movies. In
such services, the equipment’s may be operated automatically or by unskilled or
skilled labour.

b. Labour based : Here, the human element is primary in the production and
delivery of services. The equipments or machines, if any, are secondary. This
type of services includes counselling, legal advisory service, catering and hair
dressing service. There are services in which both the equipments and labour
have equal importance as in the case of hospitals.

According to Nickles services can also be classified as being convenience, shopping


and specialty services.

a. Convenience services are those which the customer usually purchases


frequently, immediately and with the minimum of efforts. Dry cleaning services,
and shoe repairing services are examples of convenience services. Convenience
of availability with minimum efforts determines the buying decisions. The user is
not prepared to go to any effort to secure a supply and will accept a substitute
often compromising on price and quality. So the marketer must secure a widest
possible availability if he is to maximize sales.

b. Shopping services are purchased often comparing quality and price. As


information regarding the service product is important for customer comparisons,
a marketing strategy has to be evolved providing enough information to the
customers. Word of mouth is also an important factor in the selection of shopping
services. Shopping services include banks, insurance companies, physicians and
beauticians.

c. Specialty services: Here the customer puts in special purchasing efforts. The
customers will be ready to travel distances and pay a premium for the services.
Specialty services include medical specialist and legal advisors. As the customer
is willing to take social purchasing efforts the marketing strategy will be focused
on service product and building customer satisfaction.

According to Hill the services may be classified as follows:

Abhishek kumar yadav 520844307 Page 53


a. Affecting person (eg. Health) or affecting goods (eg.cargo maintenance),
b. According to the permanent or temporary effects,
c. According to the reversibility or irreversibility of those effects,
d. Physical effects or mental effects and
e. Individual or collective services.

Q. 2. (3)Describe the stages involved in the positioning of services.

Ans. Positioning is concerned with the identification, development and communication of a


differentiated advantage which makes the organization’s products and service
perceived as superior and distinctive to those of its competitors in the mind of its target
customers

Services have a number of distinguishing features which have special implications for
the positioning and attributes to emphasize. Three key characteristics for service
positioning are the intangibility, the degree of variability or heterogeneity in quality of a
given service, and inseparability. Positioning can be considered at several levels such
as:

a. Industry Positioning – the positioning of the service industry as a whole.


b. Organizational Positioning – the positioning of the organization as a whole.
c. Product Sector Positioning – the positioning of a range or family of related
products and service being offered by the organisation.
d. Individual Product or Service Positioning – the positioning of specific
products.
Service positioning involves a number of steps including the following:

1. Determining Levels of Positioning: The first step in positioning is to determine


which levels(s) – service level, product sector level, corporate level-are to receive
explicit positioning attention. Some examples will illustrate the choices that are made
by some service organisations. The levels of positioning to be undertaken are
usually fairly clear cut, although some organisations have placed different emphasis
on these levels at different points of time.

2. Identification of Attributes: Once the level of positioning has been determined, it is


necessary to identify the specific attributes that are important to the chosen market
segments. In particular, the way in which purchasing decisions are made should be
considered. Individuals use different criteria for making a purchase decision of a
service.

Abhishek kumar yadav 520844307 Page 54


3. Location of Attributes on Positioning Map: The positioning process involves the
identification of the most important attributes and location of various companies’
service for these attributes on a positioning map. Where a range of attributes are
identified, the statistical procedures which exist for combining these attributes are
identified. Statistical procedures are referred to by various names such as principle
components, multi-dimensional scales, factors etc. depending upon how the data
were elicited and which statistical procedures were used.

Products or service are typically plotted on a two dimensional positioning map. The
positioning map can be used to identify the position of competitor’s service in
relation to the selected attributes. The analysis can be further developed by drawing
separate positioning maps for each market segment. Customers in each market
segment may perceive the service and its benefit differently, and different maps will
show these different positions.

Positioning maps can be based on either objective attributes or subjective attributes.


Maps can also use a combination of objective and subjective attributes.

4. Evaluating Positioning Options

 Strengthening current position against competitors to avoid head-on attack.


 Identifying an unoccupied market position that was not filled by a competitor.
 Repositioning the competition.
Once a company had identified where it is positioned at present, needs to determine
how to enhance or sustain its position relative to its competitors.

Criteria for Good Positioning are


 The positioning should be meaningful.
 The positioning must be believable.
 The positioning must be unique.
5. Implementing Positioning and the Marketing Mix: How a company and service is
positioned, needs to be communicated throughout all of its implicit and explicit
interactions with customers. This suggests that all elements of the company; its
staff, policies and image will together convey the desired position to the market
place. This means that a company must establish strategic positioning directions,
which are followed through in all of its tactical marketing and sales activities.

A successful positioning strategy should make the service clearly distinguishable by


features which are desirable and important to the target customer segment. This
means that the positioning strategy should be examined from time to time to ensure

Abhishek kumar yadav 520844307 Page 55


that it does not become outdated and that it is still relevant to the target market
segment.

Q. 3 (4)Explain in detail seven Ps of service marketing.

Ans. The marketing mix concept is a well established tool used as a structure by markets. It
consists of the various elements of a marketing programme which need to be
considered in order to successfully implement the marketing strategy and positioning in
the company’s markets. Essentially the marketing mix represents the factors which
need to be considered when determining a service firm’s marketing strategy. The
marketing strategy for a service company depends on seven Ps which are - product,
price, place, promotion, people, processes and physical evidence. The underlying
concept in developing each of these elements is to use them to support each other to
reinforce the positioning of the service product and to deliver appropriate service quality
to achieve competitive advantage.

1. Product:

The term ‘product’ is frequently used in a broad sense to describe either a


manufactured good or service. Thus, goods and services are two types of product. A
service product denotes an activity or activities that a service provider offers to perform,
which results in satisfaction of a need or want of predetermined customers.

In planning the offer of products and services, good marketing manager devices a
strategy whereby the offers are viewed at various levels to achieve unmatched product
differentiation and superlative customer service. Generally, four levels of service
products are identified which are as follows:

a. The Core or Generic Product It is the basic service product. Although the term
‘generics’ is the most usual descriptive term, the generic products have also been
described as brand free, no names and unbranded products. A typical example
would be a bed in a hotel room for a night.

b. The expected product: It is the minimum set of benefits expected by a customer


from a service product. It consists of the generic product and the minimal purchase
conditions which have to be met. Thus, a customer having a transaction with bank
will expect, in addition to the service, a correct transaction recording, timely service
and minimum courtesy.

c. The Augmented Product : They are the offerings what the customers expect
besides the benefit expected by them. The augmented product is described as the
complete bundle of attributes perceived by or offered to an individual buyer

Abhishek kumar yadav 520844307 Page 56


incorporating:

d. The Potential Product: It consists of potentially feasible added features and


benefits to hold and attract customers. It includes the potential for redefinition of
the product to take advantage of new users and the extension of existing
applications.

A service product is a complex set of value satisfactions. People buy services to solve
problems and they attach value to them in proportion to the perceived ability of the
service to do this. Value is assessed by the buyers in relation to the receivers. But it has
to be recognized that customers differ and that their requirements for different attributes
vary by market segment.

2. Pricing

Until recently, two board strategic approaches to prices were in vogue. They are (i)
Skimming and (ii) Penetration. Skimming strategy is based on the perceived need of
the users which tend to affect their sensitivity to the prices. When they are insensitive to
the prices it could be exploited by setting a very high price to skim the cream off
market. Whereas, a penetration strategy assumes that by producing a product similar
to that of a competitor and then under pricing it and thereby some or all its market
share can be taken away. The recent trend is an alternative value-based strategy
based on the belief that the appropriate concept is the perceived value held by the
customer.

Pricing Objectives: The pricing methods being adopted should consider the pricing
objective of the service firm. The most typical pricing objectives that a company could
have are as follows:

a. Pricing to achieve a desired return on investment:

b. Stabilization of pricing margin to ensure maximum profit;

c. Pricing to realize the target market share; and

d. Pricing to meet and prevent competition for survival.

These represent some of the more common, pricing objective.

The decision on pricing will be dependent on a range of factors including:

a. Corporate objective
b. Positioning of the service
c. The nature of competition

Abhishek kumar yadav 520844307 Page 57


d. Lifecycle of the service(s)
e. Elasticity of demand
f. Cost structures
g. Shared resources
h. Prevailing economic conditions
i. Service capacity
Pricing Method: After considering the demand, cost, competitors and all the other
relevant factors in the light of the pricing objective, the service marketer should consider
the method by which the prices are fixed. There are different methods and some of
them are:

a. Cost-plus Pricing: It includes all methods of setting prices with exclusive


reference to cost. By adding an amount of money to an estimated product cost a
selling price is arrived at. This money which is added is considered as the profit
expectation, if the sale is made on the basis of adding this anticipated profit to total
or full costs.

b. Competition Pricing: Pricing based on market conditions where firms compete


with one another by undercutting others prices, rather than other forms of
competition such as product quality, product differentiation and advertising.

c. Competitive Parity Pricing: Pricing is done on the basis of those that are followed
by the competitor or market leader.

d. Loss Leading Pricing: The price of a product or service is deliberately cut to a


point below its cost, aiming to attract additional customers willing to buy profitable
items. It is usually applied on a short-term basis to establish position in the market.

e. Rate of Return Pricing: It is also referred to as target return pricing. The prices
are set to achieve a given rate of return on investments and assets.

f. Value Based Pricing: It is market driven and reinforces the positioning of the
service and the benefit the customer receives from the service. In value based
pricing, prices are based on the services perceived value to a given consumer
segment.

g. Relationship Pricing: The future potential profit streams over the lifetime of
customers, forms the basis for relationship pricing. It is considered that the
relationship pricing is the appropriate form of pricing where there is an ongoing
contact between the customer and the service provider. It is said that the
relationship pricing follows closely the market oriented approach of value based

Abhishek kumar yadav 520844307 Page 58


pricing but takes lifetime value of the customer into account.

h. Prestige Pricing: The pricing above the given market price on the basis that many
buyers regard price as an indicator of quality and so will perceive enhanced quality
to products with higher than usual prices. In such cases sellers will be able to ask
prestige prices for products which have distinctive brand names and reputation.

Place & Location

The location and channels used to supply services to target customers are two key
decision areas. Location and channel decisions are essential to consider how and at
which place the services can be delivered to the customer. They become more relevant
to service as they cannot be stored and mostly are produced and consumed at the
same point. The environment in which the delivery of service takes place and the
manner of its delivery is important attributes of the service when its value is perceived.

Location : Service location is an important consideration in place strategy. A service


firm should decide where its operations and staff are situated, because if they are not
conveniently located, the customers may turn to rival service provider who, in their
perception, are conveniently placed. The type and degree of interaction is an important
factor involved in the location of services. Depending upon the nature of the service,

a. the customer may go to the service firm, or


b. the service firm may go to the customer or
c. the service provider and the customer may transact business at arm’s length.
In the first type of interaction mentioned above, where a customer goes to the service
provider, location selection becomes very important. For a service business such as a
restaurant, location may be one of the main reasons for patronage. In this type of
interaction, service at more than one location is called for.

In the second type of interaction, where the service provider can go to the customers,
site location becomes much less important, provided it is sufficiently close to the
customers for good quality service to be received. In some circumstances, the service
provider at the customers promises. This is the case with a wide range of maintenance
services such as lift repair, pest control and cleaning services. In other cases, service
providers have discretion in whether they decide to offer their services at the customer’s
home or office, as do hairdressers and TV repair firms. Some dry cleaning and laundry
firms have built up profitable businesses by dispensing with the need for expensive
multiple high street locations and locating their operations in a low cost area and
providing a pick-up and delivery service.

However, when the customer and service organization transact at arm’s length, location

Abhishek kumar yadav 520844307 Page 59


may be largely irrelevant. The customers are not concerned with the physical locations
of suppliers of service such as electricity, telephone or insurance, provided efficient mail
or electronic communications are in place.

Some of the critical factors affecting the location decision are:

a. Convenience perceived by the customer,


b. Operating cost,
c. Comparative proximity with that of the competitors,
d. Geographic and supporting systems,
e. Geographic and environmental factors,
f. Business climate in the particular location,
g. Availability of communication networks,
3. Promotion and Communication

Promotion is a set of activities designed to increase by consumers and is the through


which the service provider communicates with his target markets. The promotion of
services covers a number of areas or promotional tools which form the communications
mix or promotions mix. These include:

a. Advertising

b. Personal selling

c. Sales promotion

d. Public relationship

e. Word of mouth, and

f. Direct mail.

The use of promotional tools in service sector is a comparatively recent aspect. Now,
the fiercely competitive environment has made the service marketers use promotional
tools actively. But promotional exercise as it is done in consumer goods companies is
not the same for service industry.

A communication programme for a service organization may consist of a wide variety of


alternative communications and promotion. To communicate the target markets, the
various elements of the communication mix must be integrated within the promotion
and communication programme. This process involves many tasks among which the
following are considered important.

a. Identification of target audience,

Abhishek kumar yadav 520844307 Page 60


b. Determination of promotion objectives,
c. Selection of communications mix.

The choice of the communications mix for service involves decisions on such issues as
whether to advertise, use personal selling or generate publicity through greater public
awareness by such means as through editorials, publications and press activity. The
choice of medium is determined by decisions on how to create the most favourable
awareness amongst the target audience.

Advertising is one of the main forms of impersonal communication used by service


firms. The role of advertising in service marketing is to build awareness of the service,
to add to the customer’s knowledge of the service from other service offerings. Relevant
and consistent advertising is therefore of great importance to the success of the
marketing of the service.

Selection of appropriate media and determining the balance between them is essential
to obtain the most effective return on advertising expenditure. A consideration of the
specific advertising goals to be accomplished will facilitate this process.

4. People

The success of marketing a service is tied closely to the selection, training, motivation
and management of people. There are many examples of services failing or succeeding
as a consequence of the ineffective or effective management of people.

All the people participating in the delivery of service provide cues to the customer
regarding the nature of the service itself. How these people are dressed up, their
personal appearance and their attitudes and behaviours influence the customer’s
perceptions of the service.

Kotler says that, “if the service personal are cold or rude, they can undermine all the
marketing work done to attract customer. If they are friendly and warm, they increase
customer satisfaction and loyalty”. Hence, the importance of people within the
marketing of services has gained much interest in internal marketing.

In turn, the company made employees feel wanted and cared for, building on the
principle that those who are looked after will pass on this caring attitude. The success of
this new direction for the airline brought increased profits matched by greater customer
and employee satisfaction.

5. Process

The process by which the service is created and delivered to the customer is critical to

Abhishek kumar yadav 520844307 Page 61


the service operations as customer often perceive the service delivery system as part of
the service itself.

Process means all work activities. Process involve the procedures, tasks schedules,
mechanisms, activities and routines by which a product or service is delivered to the
customer. It involves policy decisions about customer involvement and employee
discretion. The customer not only thinks about the service product alone but also
attaches importance to the manner in which it is delivered. Under these circumstances,
a poorly designed service process leads to poor service quality. Banks provide a good
example of this. By reconfiguring the way they deliver service through the introduction
of automatic teller machines (ATMs) banks have been able to free staff to handle more
complex customer needs by diverting cash only customers to the ATMs.

Processes are seen as structural elements that can be engineered to help deliver a
desired strategic positioning. They can be analyzed according to the complexity and
divergence. Processes can be changed to reinforce the positioning. A clear
understanding of the configuration processes in terms of this complexity and
divergence, on a balance of marketing and operations activities are important factors for
improving service systems. Processes are thus a marketing mix element which can
have a substantial role in reinforcing positioning and in product development.

7. Physical Evidence

The environment in which the service is delivered and where the firm and customer
interact, and any tangible components that facilitate performance or communication of
the service is known as physical evidence in service.

The physical evidence of service includes all of the tangible representations of the
service such as brochures, letterhead, business cards, report formats, signage, and
equipment. In some cases, it includes the physical facility. Physical evidence cues
provide excellent opportunities for the firm to send consistent and strong messages
regarding the organization’s purpose, the intended market segments and the nature of
the service.

Abhishek kumar yadav 520844307 Page 62


SERVICE MARKETING AND CUSTOMER RELATIONSHIP
MANAGEMENT

(MK-0006)

ASSIGNMENT-2

Q. 1 1. Explain the tourism marketing mix.

Ans. Tourism marketing is a process of creating a product or providing a service. It


comprises fact finding, data gathering, analyzing, communication to inform and
promote, ensuring and facilitating sales, selection of marketing planning, coordination,
control and evaluation, developing professionally sound personnel.

Tourism marketing is an integral effort to satisfy tourist and more so, it is a device to
transform the potential tourists into the actual tourists. Tourism marketing is the safest
way to generate demand, expand market and increase the market share. The concept
of tourism marketing comprises of:

a. Identifying and anticipating consumer demand and desire for tourism products and
services.

b. Developing a means of providing product and services to fulfill these needs.

c. Communicating this to the consumer, thereby motivating sales, consequently


satisfying both the consumer, and the organizations objectives.

Through market planning, segmentation and marketing research, a tourism marketing


mix can be developed to achieve the tourism origination goals through strategic
marketing.

The tourism Marketing Mix

The marketing mix refers to the blend of ideas, concepts and features which marketing
management put together to best appeal to their target market. Segments, each target
segment will have a separate marketing mix, tailored to meet to specific needs of
consumers in the individual segment.

1. Product Mix :

The tourism product is a non material intangible thing. The following services are
included in the product mix for the tourism industry.

a. Attraction

Abhishek kumar yadav 520844307 Page 63


b. Accommodation: one star to five star, cottages.
c. Transportation: Airways, Railways, Road, Sea Ways.
d. Recreation: Theatre, Club, Park, Music
e. Restaurant : western, Chinese, Indian others.
f. Shopping : Artistic, handicrafts, handloom, books, Garments, jewelry.

Branding plays a very important role in tourism marketing. Car rental firms, hotel chains
and airlines, in particular employ tremendous efforts to ensure that their name is widely
recognized and synonymous with quality, value or some other characteristic. Travel
agents and tour operators depend on reputation to a large extent, and so it is
imperative that they have a strong, recognizable identity. The main reason to build
brand loyalty is to encourage repeat business.

2. Price Mix

In the tourism industry, the pricing decisions are found critical and challenging since it
is a multi segment industry. When a tourist proposes to visit a particular place, the total
cost on his / her traveling includes the expenses incurred on transportation,
accommodation, communication or so.

In the pricing decisions, the product or the service mix of the tourist organizations is
found important. This makes it essential that the tourist organizations set prices in line
with the quality of services to be made available to the customers.

The tourist professionals while making the pricing decisions are required to think in
favor of discounting price. The different forms of discount, such as discounting for cash
payment, price reduction for quality, trade discounts, trade allowances, seasonal
discount, distressed stock and similar discount tactics are the options.

3. Place Mix

Distribution management is concerned with two things availability accessibility. If


tourism marketing management is to be certain that their products and services are
both available and accessible to the target market, they must design a channel strategy
that will be effective.

Some tourist organizations deal directly with the consumers; some other organizations
utilize more than one method distribution. The middlemen are the tour operators and
the transport operators who buy services like hotel rooms, seats in the aircrafts,
railways, arrange chartered flights and sell the same either to the travel agents or even
directly to the tourists. Airlines, for example, sell tickets through travel agents, and sell
seats on flights to tour operators, whilst also operating direct marketing by offering

Abhishek kumar yadav 520844307 Page 64


travelers the opportunity to make reservations through their own booking officers.

4. Promotion Mix

The tourist organization takes up the responsibility of informing, sensing and


persuading the potential tourists. The marketers need to use the various components
of promotion to increase the number of users.

The advertisement helps in providing important information to the actual and potential
tourists. Its coverage is so wide. It essentially follows the AIDA principle of attracting
the attention, arouse internet, create desire, and stimulate action. Advertising is aimed
to create the awareness of the travel offers available on a resort and its attraction to
influence their business decisions.

Sales promotion through brochures, point of sale displays and even video cassettes
plays a very important role as advertising. In a tourism industry a travel company offers
to their client’s compliments such as flight bags, wallets for tickets and foreign
exchange and covers of passport. The hotels offer a number of facilities like shoe shine
clothes, first aid sewing kits and shampoo, further, the clients also get fruits and flowers
in their rooms. There is no double that almost all the promotional measures generates
goodwill and add value to the product.

5. Personal selling:

The travel and hotel business depend considerably on the personal selling. The
developing of travel and tourism has been possible due to well educated and trained
sales personnel.

6. People:

The tourism industry depends substantially on the management of human resources.


The travel agents and travel guides plays an important role and therefore the
management of people helps in developing their credentials to deliver goods to the
tourist organizations.

Q. 2 3. Write a note on maximizing the customer value through CRM.

Ans. Customer Relationship Management (CRM) is the relationship management of a


customer by an organization. CRM means “the establishing, the developing, the
strengthening, and the trusting of good values among the customer for company’s
brand.”

Adopting CRM is business critical decision. If a company does not develop


synchronized multi – channel customer interface capabilities, it may soon lose

Abhishek kumar yadav 520844307 Page 65


customers to competitors. The age of “build a better mouse trap and the customers will
come” is over. The experience of buying the mousetrap can be as important as the
product itself.

Here are key imperatives for organisations seeking to build a true e-CRM

a. Focus on Profit : A com. customer strategy focuses on customer acquisition. A


Profit strategy focuses on customer retention and profitability as well.

b. Design the total customer experience, not just the interface. Devote specialize
resources – ergonomic engineers, psychologists, focus groups and testers – to
design the total customer experience.

c. Profile customers for profitable segmentation. Intensively segmentation base to


learn about your best customer to develop special programs that remain them and
reward them with the perks they deserve. Take action to bring unprofitable
customers into the black.

d. Embed business logic into your infrastructure. Business logic is the system of
procedures and rules that underlie how the CRM system operates its technology
and the people with the customers.

e. Opportunistically outsource. Get to market with CRM capabilities faster and without
huge capital commitments and old-style systems development efforts. Implement
disposable infrastructure applications that can be added cheaply and then thrown
away.

f. Manage multi-channel performance. The performance of your company as


experienced by customer must be consistent regardless of which channel they
choose to work through. Measure performance rigorously first to understand which
channels need work and second to provide continuous feedback across multiple
channels.

g. Be trustworthy with customer’s information.

h. Staff and train for e-business.

i. Learn from the experience of ERP.

CRM has gained importance in the recent days. Developing appropriate strategy is
critical decision of a company. CRM uses the electronic media to operate and simplify
customer related business processes, reducing the cost of customer facing operations.

Abhishek kumar yadav 520844307 Page 66


Q. 3 5. Describe the use of CRM in designing and delivering the services.

Ans. CRM applications, often used in combination with data warehousing, E-commerce
applications, and call centres, allow companies to gather and access information about
customers’ buying histories, preferences, complaints, and other data so they can better
anticipate what customers are looking for. The other business objectives include:

a. Increased efficiency through automation

b. The ability to provide faster response to customer inquiries

c. Having a deeper knowledge of customer needs

d. Generating more marketing or cross-selling opportunities

e. Better information for better management

f. Reduced cost of sales and increased productivity of Sales Representatives

g. Receiving customer feedback that leads to new and improved products or


services

h. Conducting more one-to-one marketing.

Essence of a CRM solution

In many companies, sales, marketing and customer service/support organizations work


as decentralized functions. This forces customers to run from pillar to post when trying
to meet their demands, creating a good deal of dissatisfaction. CRM provides a
common platform for customer communication and interaction. The use of CRM
applications can lead to improved customer responsiveness and a comprehensive view
of the entire Customer Life Cycle. It can extend the traditional channels of interaction
such as direct sales force or tele-business to the Web by providing a framework for
managing the interactions and transactions. It also enables the customers to purchase
products or services on-line and receive web-based services and support; with
everything personalized to the individual customer.

Use Of CRM in Designing and Delivering of Services:

Currently, the CRM market is dominated by front office automation applications.


However, CRM system users realize that CRM applications are not providing an
enterprise-wide view of the customer. Because of this, today CRM embraces a range
of technologies including Data Warehousing, ERP and SCM applications.

The integration framework is becoming even larger as a result of the web-based


initiative. In fact, the web will become so important that analysts feel it may overshadow
the category as a whole. Stan Dolberg, a Senior CRM Analyst at Forrester Research

Abhishek kumar yadav 520844307 Page 67


Inc., Cambridge, calls CRM a dead end that will be replaced by Enterprise Relationship
Management (ERM) – a class of applications that uses the Web to place the Customer
at the centre of trading relationships.

The ongoing consolidations and mergers across the ERP, CRM and other technology
vendors further highlight this point. (Vantive has been taken over by PeopleSoft and
Clarify has merged with Nortel).

ERM, which takes CRM to the next level, is the current hot topic of the industry. CRM
automates certain functions in certain departments within the organization.

ERM attempts to align the entire enterprise to provide a single view to the customer.
The ERM technology framework will generate a universal business application covering
everything from CRM to ERP, and SCM applications as well as Data Warehousing and
OLAP. It will present a cohesive set of analytical models that will take into the account
the cross departmental functions and interdependencies.

Maintaining relationship by using CRM models

Most companies truly believe they are customer-focused, but in fact they are still very
product centred – they still develop products and services and then search for
customers. The challenge, then, is to shift their thinking to a more customer-focused
world – where they create new products and services that follow the customer’s lead.
To make that shift, however, companies will have to change their core-operating
model. The biggest challenge about implementing a CRM application is that to ensure
you have a system, which is able to reach to the existing as well as potential customers
and cater to their individual needs.

Abhishek kumar yadav 520844307 Page 68


ADVERTISING MANAGEMENT & SALES PROMOTION

(MK-0007)

ASSIGNMENT-1

Q. 1 Describe an ad that you have seen recently. Analyze the ad in terms of its
effectiveness in generating Attention, Interest, Desire and Action as per the AIDA
model.

Ans. AIDA is an acronym used in marketing that describes a common list of events that are
very often undergone when a person is selling a product or service:

 A - Attention (Awareness): attract the attention of the customer.

 I - Interest: raise customer


interest by focusing on and
demonstrating advantages
and benefits (instead of
focusing on features, as in
traditional advertising)

 D - Desire: convince
customers that they want and
desire the product or service
and that it will satisfy their
needs.

 A - Action: lead customers towards taking action and/or purchasing.

Marketing today allows a diversty of products. Using a system like this, allows a
general understanding of how to target a market effectively. A.I.D.A however is a
acronym that is necessary to learn in marketing.

Recently I have seen the ad of film “PAA”. The analysis of this ad as per AIDA model is
as below:

Abhishek kumar yadav 520844307 Page 69


A – Awareness: It demonstrated that movie goers to be aware of a product's
existence ie the new movie. R Balkrishnan, the director of the film and also the brain
behind its promotion campaign wanted to avoid the usual “razzmatazz”, he wanted it to
be executed in such a way, that people discuss Auro the character and not Amitabh
the.. Teaser ads’ was an innovation, very efficiently used in the campaign. A teaser ad
showing a bald head in a card board box increased the curiosity of the people which
multiplied exponentially, as it became the topic of discussion.

In Balki’s words, “The noise level has beaten our expectations.”

Big B’s decision to visit schools to bring about awareness regarding the disease is the
ace up his sleeve. This will contribute heavily in the promotion of the film.

With so much new coming up by ways of innovation, how could the titles lag behind.
Well for the first time in the history of Bollywood, the cast and the credits of the film will
be introduced by Jaya Bacchhan, cutting out the usual trends of titles.

And last but not the least, it is for the first time that Aishwarya Bacchhan is involved in
the marketing and promotion of a film.

With discussions on end with the marketing and creative heads of the film, she is
exploring every possible strategy, for maximum interaction with the people

A highly interactive website was just another step that has paid high dividends.
Paathefilm.com has received innumerable responses from the time it was created.

Not to mention his blogs which have become a very powerful platform for Amitabh
Bacchhan, which he has been using very effectively for the promotion of “Paa”.

It is these experiments with innovations which have made Paa the talk of the country.
Paa is a 'rare' story about a father-son, son-father relationship.

I – Interest: Raised the movie goers interest by iintroducing the hoardings all around
the city created an interest in the movie goers Also the tagline Paa is a 'rare' story
about a father-son, son-father relationship. Abhishek Bachchan playing the role of
father to Amitabh Bachchan , the biggest actor India has ever produced. A new getup,

Abhishek kumar yadav 520844307 Page 70


a new look, a totally transformed Amitabh. He endured a lot for that. From shaving off
his body hair, to sitting for 5 hours at a stretch to get the makeup on and then
struggling for 2 more hours to get it off. starAuro (Amitabh Bachchan) is an intelligent,
witty 13 yr old boy with an extremely rare genetic defect that causes accelerated
ageing.

He suffers from a progeria like syndrome. Mentally he is 13, very normal, but physically
he looks 5 times older. In spite of his condition, Auro is a very happy boy. He lives with
his mother Vidya (Vidya Balan), who is a gynaecologist.

Amol Arte (Abhishek Bachchan), is young, progressive and a full of ideals, politician.
He is out to prove to the world that 'politics' is not a bad word. He is a man with a
mission. Auro is Amol's son. The promotion of Amitabh Bachchan in a extremely
different role and the hoardings all around the city created an interest in the movie
goers . The details about the disease which people have not heard created a feeling of
insecurity and public tried to find it out from internet asked doctors

D-Desire : A fresh movie with a different theme. an extremely rare genetic defect that
causes accelerated ageing.He suffers from a progeria like syndrome. Mentally he is 13,
very normal, but physically he looks 5 times older.A new story line and it created ripples
in the movie Box Office.

A-Action: A new story line and it created ripples in the movie Box Office. The people
liked the movie. The producer and director of the film were able to pull the people out of
the homes to go and see the movie, Got 3– and 4–star ratings and publicized that in
promos.

Q. 2 Select an ad of your choice and describe its creative execution. What is the
advertising strategy behind the ad in terms of a) Objective b)Target audience c)
Key consumer benefit d) Reason to believe e) Proof and f) Tone and manner?

Ans. I select ad of Vodafone mobile services for the discussion. 'Zoozoos', a new innovative
concept from mobile service provider Vodafone Essar is on its way to capture hearts of
millions of viewers similar to that of 'Hutch dog'.

Zoozoos -- stick-like figures with an egg-like heads -- feature in the latest Vodafone
Essar commercials that are aired during the Indian Premier League Twenty20 cricket
series to promote different offerings and services provided by Vodafone to its
customers.

Thes characters are named Zoozoos, and are created by O&M. These ads were
directed by Prakash Varma and were launched during the ongoing IPL 2 series.

Abhishek kumar yadav 520844307 Page 71


Zoozoos aren’t Animated Characters, they are real people

The Zoozoo characters that appear in the Vodafone ads are not animated characters
but are real actors from Mumbai and Capetown theaters. The actors wore special
costumes to portray Zoozoos to create an illusion of animation. The costume is divided
into two parts, the head and the body. The body part of the costume is stuffed with
foam to create rounded shapes.

To create large heads they used a material called Perspex, which was again stuffed
with foam. Also, they casted only women and children to play the Zoozoos so that the
thin hands and legs made the heads look even bigger.

These ads were shot at a very slow frame rate to make the movements look funny.

Animation requires so much detailing and here we had to do the exact opposite. We
had to make real characters look like animated characters. It was quite challenging as
none of them could see as they were covered from head to feet.

It is not for nothing that Indian advertising is seen as coming into its own. And
Vodafone advertising campaigns have been at the forefront of it all.

First there was that dog in Hutch campaigns, which captured the imagination of
millions. It survived a $19-billion buyout and reached such levels of popularity that it
even led to kidnappings of that breed of dogs, called pugs.

And now come those nutty and entirely endearing little men in white, called Zoozoos
created by the agency O&M for Vodafone, one of the main broadcast sponsors of the
Indian Premier League. They are wowing audiences, and ZooZoo merchandise, bags,
keychains, T-shirts will be available soon.

In about 10 days, O&M completed the campaign shooting two-three films a day, each
selling a product or a service offered by Vodafone. A dozen more films are expected as
the league progresses.

Abhishek kumar yadav 520844307 Page 72


The characters, which look like distant cousins of the Pillsbury dough boy, were
enacted by professional ballet artists in white body suits. What made them so
endearing was that they are innocent people living in a simple world unlike ours, who
laugh loud when they laugh. And who seem to be in an in-between world of animation
and reality.

The strategy is generating a buzz that lives up to the brand’s image of great creative
and clever marketing. The company spends about Rs 250 crore on advertising across
mediums according to industry sources.

The details of the advertisement are as follows:

1. Objective: To make the Vodafone company No.1 in the country .

2. Target Audience: To target the Indian customer during IPL Season-2. The Indian
middle class is obsessed to mobile phones and cricket. The Vodafone company
could not have asked for better time to launch an advertisement.

3. Key consumer benefits :The each selling a product or a service offered by Vodafone.
was shown each day during the match. Creating an image in the mind of audience.
The advertisement also showed the benefits in hilarious animated type form making
it more interesting Reason to believe:.

4. Reason to believe :The Zoozoo characters that appear in the Vodafone ads are not
animated characters but are real actors from Mumbai and Capetown theaters. The
actors wore special costumes to portray Zoozoos to create an illusion of animation.
The costume is divided into two parts, the head and the body. The body part of the
costume is stuffed with foam to create rounded shapes. Animation requires so much
detailing and here we had to do the exact opposite. We had to make real characters
look like animated characters. It was quite challenging as none of them could see as
they were covered from head to feet.

5. Proof: The characters, which look like distant cousins of the Pillsbury dough boy,
were enacted by professional ballet artists in white body suits. What makes them so
endearing is that they are innocent people living in a simple world unlike ours, who
laugh loud when they laugh. And who seem to be in an in-between world of
animation and reality,

6. Tone and Manner : The tone was soft and manner in which it was presented is really
fantastic. It showed a new face of Indian Advertising.

Q.3 Prepare an outline of a media plan for a newly launched soft drink. Include
a)Media objective(s) b)Media mix c) Allocation of budget among media mix and

Abhishek kumar yadav 520844307 Page 73


d) Media scheduling.

Ans. In order to develop a media plan for launching a new soft drink one has to:

1. Media Objectives : The largest category in an advertising budget is the media


costs--the money one spends. The manager has to organize the day-to-day tasks of
carrying out the strategies. The tool needed to do this is a media plan that begins
with an overview and works its way down to the details. It will helps with every phase
of your advertising. The best way to go to the public for soft drink advertisement is
target audience which portion of the age group. The children, school goers, college
goers are the ones who takes the cold drinks. One must check for this. Which
medium to go and how much using TV, Radio and magazines. For air time on radio
or for ad space in newspapers, magazines, and more. Because of this, it makes
sense to have a sound plan to manage that investment. The goals must be set. Print
advertising prices are based on the circulation of the publication in question.
Publications will quote you a circulation figure based on paid subscribers. The
audited circulation figures are verified by monitoring organizations. The publications
will try to convince you that actual circulation is higher by including the free copies
they distribute and the pass-along readership they claim. Sometimes these claims of
"bonus" circulation are valid--for example, magazines distributed on airlines get at
least eight readers per copy. Audience is the equivalent of circulation when you're
talking about broadcast media. Audience size varies throughout the day as people
tune in and tune out. Therefore, the price for advertising at different times of day will
vary, based on the audience size that the day-part delivers.

2. Media Mix : Media planning is the process of choosing a course of action. Media
planners develop yearly plans that list each media outlet--print or broadcast.
Planning then gives way to buying, as each separate contract is negotiated, then
finalized. Penetration is related to circulation. Penetration describes how much of the
total market available you are reaching. If you are in a town with a demographic
count of 200,000 households, and you buy an ad in a coupon book that states a
circulation of 140,000, you're reaching 70 percent of the possible market--high
penetration. If, instead, you bought an ad in the city magazine, which goes to only
17,000 subscribers (households), your penetration would be much less--8.5 percent.
What degree of penetration is necessary for you depends on whether your strategy
is to dominate the market or to reach a certain niche within that market.

3. Allocation of Budget among media mix: The largest category in your advertising
budget is likely to be your media costs--the dollars you spend for air time on radio or
for ad space in newspapers, magazines, and more. Because of this, it makes sense

Abhishek kumar yadav 520844307 Page 74


to have a sound plan to manage that investment. You'll want to set goals. You'll want
to describe strategies for achieving them. You'll have to organize the day-to-day
tasks of carrying out the strategies. The tool you'll need to do this is a media plan
that begins with an overview and works its way down to the details. It will help you
with every phase of your advertising. Set the minimum and maximum budget
constraints.

4. Media Scheduling: Reach and frequency are key media terms used more in
broadcast than in print. Reach is the total number of people exposed to a message
at least once in a set time period, usually four weeks. (Reach is the broadcast
equivalent of circulation, for print advertising.) Frequency is the average number of
times those people are exposed during that time period. To make reach go up, you
buy a wider market area. To make frequency go up, you buy more ads during the
time period. Usually, when reach goes up, you have to compromise and let
frequency go down. You could spend a lot of money trying to achieve a high reach
and a high frequency. The creative part of media planning comes in balancing reach,
frequency, and budget constraints to find the best combination in view of your
marketing goals.

Abhishek kumar yadav 520844307 Page 75


ADVERTISING MANAGEMENT & SALES PROMOTION

(MK-0007)

ASSIGNMENT-2

Q. 1 Describe an ad which you feel is not so effective in terms of its creative


execution. Give suggestions on how to make the ad more effective.

Ans. LUX ad is one which I don’t like as it is a illogical. The ad depicts one male model
running after a female(wife) who baths with LUX soap. Due to its and softness due to
use of the soap and fragrance the male tries to hug her and in the next scene it is
shown that the girl slips after he catches her. Next clipping shows that the male lead
model hugs some other fat lady who does not uses LUX soap.

I consider the depiction in poor sense as this is against out moral values of Indian
society. It gives a poor taste of the thinking of director of the ad film.

The visual content must show the true realities rather than fantasy. Instead of other
lady it could have been some dirty male. The creativity is missing in the ad and may be
it is a copy of some advertisement of western country.

Advertisements for products whose advertising is prohibited or restricted by law or by


this Code must not circumvent such restrictions by purporting to be advertisements for
other products the advertising of which is not prohibited or restricted by law or by this
Code. In judging whether or not any particular advertisement is an indirect
advertisement for product whose advertising is restricted or prohibited.

Following would be my suggestion to make this ad more effective:

1. Visual content of the advertisement must depict only the product being advertised
and not the prohibited or restricted product in any form or manner.

2. The advertisement must not make any direct or indirect reference to the prohibited or
restricted products.

3. The advertisement must not create any nuances or phrases promoting prohibited

Abhishek kumar yadav 520844307 Page 76


products’

Q. 2 What are some of the advantages of Advertising as compared to Sales


promotion? Which types of products lend themselves to sales promotion?

Ans. Sales Promotion: A total business communications strategy includes advertising,


sales promotion and personal selling. The cohesiveness and effectiveness of these
efforts is what achieves sales and profit objectives. Promotional strategy is the function
of informing, persuading, and influencing a consumer decision. Following are the
functions of sales promotion:

1. To provide information – In the early days of promotional campaigns, when many


items were often in short supply, most advertisements were designed to tell the
public where they could find a product. Today, a major portion of U.S. advertising is
still informational. Promotional campaigns designed to inform often target specific
market segments.

2. To differentiate – Marketers often develop a promotional strategy to differentiate


their goods or services from those of competitors. This strategy is called positioning.
The idea is to communicate to customers meaningful distinctions about the
attributes, price, quality, or usage of a good or service. Market research is a valuable
tool for positioning since it helps to identify what consumers want and what attributes
are important to them.

3. To increase sales – Increasing sales volume is the most common objective of a


promotional strategy.

4. To stabilize sales – Advertising is another tool that can stabilize sales. A stable
sales pattern has several advantages: it evens out the production cycle, reduces
some management and production costs, and makes it easier to do financial,
purchasing and market planning.

5. To accentuate the product’s value – Some promotional strategies are based on


factors that add value, such as warranty programs and repair services.

Advertising: Advertising is a paid, non-personal sales communication usually directed


at a large number of potential buyers. Types of advertising include:

1. Informative advertising – Advertising approach intended to build initial demand for


a good or service in the introductory phase of the product life cycle.

2. Persuasive advertising – Used in the growth and maturity stages of the product life
cycle to improve the competitive status of a product, institution or concept.

Abhishek kumar yadav 520844307 Page 77


3. Comparative advertising – Persuasive advertising approach in which direct
comparisons are made with competing goods or services.

4. Reminder-oriented advertising – Method used in the late maturity or decline states


of the product life cycle that seeks to reinforce previous promotional activity by
keeping the name of the good or service in front of the public.

The following are some of the most popular forms of advertising media:

1. Newspapers – Can be costly so you want to reach the exact audience that will buy
your product or service. Avoid using small print if possible. You may be able to place
an ad in the more affordable weekly papers where you can run your ad by zip code.

2. Television and radio – Are typically expensive. The most popular stations are
typically expensive. Be sure to know your target audience and study the media kits
to determine if the station reaches that audience.

3. Direct mail – Can be either generated by you individually or can be a part of a co-op
program such as Val-Pak.

4. Magazines and trade journals – Many have space available regionally.

5. Outdoor advertising including billboards and transit ads (buses, cabs)

6. Yellow pages – This is possibly the first type of advertising you should purchase. A
large ad is not necessary; a listing is sufficient to let your potential customer know
you are a valid company, not a fly-by-night. The downfall with Yellow page
advertising is that it takes the customer directly to your competition! Have a listing
but be careful while promoting it.

7. Internet – Website or banner advertising.

Some lower-cost advertising opportunities include co-op advertising programs where


there is a cost sharing arrangement between the manufacturer and the retailer, cable
TV advertising, and targeted direct mail postcards.

Q. 3 What are the differences and similarities between social awareness advertising
and advertising of products and servives, based on what is mentioned in the
case.

Ans. Social Awareness is consciousness shared within a society. It can also be defined as
social awareness; to be aware of the problems that different societies and communities
face on a day-to-day basis; to be conscious of the difficulties and hardships of society.
There is debate as to exactly what the term means. Some people define social

Abhishek kumar yadav 520844307 Page 78


consciousness as a society's consciousness of itself. Others argue against this
definition, saying that society does not have a mind of its own, and therefore is not
conscious: rather, the people that make up society are individually conscious. Social
consciousness is similar to collective consciousness. Many studies have been done to
examine the roots of social consciousness. It is believed to arise as a response to
social injustice experienced by the individual or in the lives of others around the
individual.

There are three levels of social consciousness - acquired, awakened, and expanded. A
subject with an acquired social consciousness derives his or her viewpoint from the
mainstream culture. The person with an acquired social consciousness does not
question mainstream viewpoints, and acts accordingly, without confrontation. A subject
with an awakened social consciousness explores alternatives to the dominant cultural
viewpoint. This person might identify with a marginalized group, but the mainstream
culture is central to his or her questioning or exploration. The subject recognizes and
challenges social injustice. The person actively resists power and authority. The focus
of discontent and action is often over the right to be visible, to have choice, or to be
self-determining subject with an expanded social consciousness strongly identifies with
their marginalized group. This person views status as a continuously changing social
construct, thus viewing responses as a lifelong process. This individual has an
understanding of the complexity of the social hierarchy, and acts carefully after
weighing both sides.

Social consciousness brings moral implications. Often, people with an awakened social
consciousness become socially active. A socially conscious person tends to be
empathetic towards others regardless of race, gender, ethnicity, disability, class, or
sexual identity.

Advertising is a form of communication used to influence individuals to purchase


products or services or support political candidates or ideas. Frequently it
communicates a message that includes the name of the product or service and how
that product or service could potentially benefit the consumer. Advertising often
attempts to persuade potential customers to purchase or to consume a particular brand
of product or service. Modern advertising developed with the rise of mass production in
the late 19th and early 20th centuries.

Commercial advertisers often seek to generate increased consumption of their


products or services through branding, which involves the repetition of an image or
product name in an effort to associate related qualities with the brand in the minds of
consumers. Different types of media can be used to deliver these messages, including
traditional media such as newspapers, magazines, television, radio, billboards or direct

Abhishek kumar yadav 520844307 Page 79


mail. Advertising may be placed by an advertising agency on behalf of a company or
other organization.

Organizations that spend money on advertising promoting items other than a consumer
product or service include political parties, interest groups, religious organizations and
governmental agencies. Non-profit organizations may rely on free modes of
persuasion, such as a public service announcement.

Advertising is communication used to influence individuals to purchase products or


services or support political candidates or ideas. Advertising can be displayed on
billboards, newspapers, T.V., websites, movies and more.

Abhishek kumar yadav 520844307 Page 80


E-MARKETING

(MK0008)

ASSIGNMENT-1

Q. 1 How does e-marketing relate to e-business?

Ans. E-Marketing: E-marketing, also referred to as online marketing or internet marketing, is


marketing that uses the Internet. The Internet has brought many unique benefits to
marketing, including low costs in distributing information and media to a global
audience. The interactive nature of Internet media, both in terms of instant response,
and in eliciting response at all, are both unique qualities of Internet marketing. E-
marketing ties together creative and technical aspects of the internet, including design,
development, advertising and sales. E-marketing methods include search engine
marketing, display advertising, e-mail marketing, affiliate marketing, interactive
advertising and viral marketing.

E-Marketing can also be defined as a subset of e-Business that utilizes electronic


medium to perform marketing activities and achieve desired marketing objectives for an
organisation. Internet Marketing, Interactive Marketing and Mobile Marketing for
example, are all a form of e-Marketing.

E-Business: E-Business means utilizing, electronic medium in every day business


activities. There are several levels of involvement in when it comes to e-Business. For
example where one organisation relies completely on e-Business the second one may
choose a mixed presence and means of doing business.

Difference between e-Business and e-Marketing

E-Business is a very broad entity dealing with the entire complex system that
comprises a business that uses electronic medium to perform or assist its overall or
specialized business activities.

E-Commerce is best described in a transactional context. So for example an electronic


transaction of funds, information or entertainment falls under the category handled by
principles of e-Commerce. Technically e-Commerce is a part of e-Business.

E-Marketing is also a part of e-Business that involves electronic medium to achieve

Abhishek kumar yadav 520844307 Page 81


marketing objectives. E-Marketing is set on a strategic level in addition to traditional
marketing and business strategy.

Relation of E-Marketing with E-Business

Electronic Business, or "E-Business", may be defined broadly as any business process


that relies on an automated information system. Today, this is mostly done with Web-
based technologies. Electronic business methods enable companies to link their
internal and external data processing systems more efficiently and flexibly, to work
more closely with suppliers and partners, and to better satisfy the needs and
expectations of their customers. In practice, e-business is more than just e-marketing.
While e-business refers to more strategic focus with an emphasis on the functions that
take place using electronic capabilities, E-marketing is a subset of an overall e-
business strategy. E-marketing seeks to add revenue streams using the World Wide
Web or the Internet to build and enhance relationships with clients and partners and to
improve efficiency using the Empty Vessel strategy. Often, E-marketing involves the
application of knowledge management systems.

E-business involves business processes spanning the entire value chain: electronic
purchasing and supply chain management, processing orders electronically, handling
customer service, and cooperating with business partners. Special technical standards
for e-business facilitate the exchange of data between companies. E-business software
solutions allow the integration of intra and inter firm business processes. E-business
can be conducted using the Web, the Internet, intranets, extranets, or some
combination of these.

Q. 2 Distinguish between e-Marketing and Traditional marketing.

Ans. E Marketing Vs. Traditional Marketing

When implemented correctly, the return on investment (ROI) from


e-Marketing can far exceed that of traditional marketing strategies. Whether you’re a
"bricks and mortar" business or a concern operating purely online, the Internet is a
force that cannot be ignored. It can be a means to reach literally millions of people
every year. It’s at the forefront of a redefinition of way businesses interact with their
customers.

The benefits of e-Marketing over traditional marketing are as follows:

1. Reach: The nature of the internet means businesses now have a truly global reach.
While traditional media costs limit this kind of reach to huge multinationals,
eMarketing opens up new avenues for smaller businesses, on a much smaller

Abhishek kumar yadav 520844307 Page 82


budget, to access potential consumers from all over the world.

2. Scope: Internet marketing allows the marketer to reach consumers in a wide range
of ways and enables them to offer a wide range of products and services. E-
Marketing includes, among other things, information management, public relations,
customer service and sales. With the range of new technologies becoming available
all the time, this scope can only grow.

3. Interactivity: Whereas traditional marketing is largely about getting a brand’s


message out there, eMarketing facilitates conversations between companies and
consumers. With a two-way communication channel, companies can feed off of the
responses of their consumers, making them more dynamic and adaptive.

4. Immediacy: Internet marketing is able to, in ways never before imagined, provide an
immediate impact. Imagine you’re reading your favourite magazine. You see a
double-page advert for some new product or service, maybe Samsung’s latest
plasma TV or Apple’s latest iPod offering. With this kind of traditional media, it’s not
that easy for you, the consumer, to take the step from hearing about a product to
actual acquisition.

With e-Marketing, it’s easy to make that step as simple as possible, meaning that
within a few short clicks you could have booked a TV or ordered the iPod. And all of
this can happen regardless of normal office hours. Effectively, Internet marketing
makes business hours 24 hours per day, 7 days per week for every week of the
year.

5. Demographics and targeting: Generally speaking, the demographics of the


Internet are a marketer’s dream. Internet users, considered as a group, have greater
buying power and could perhaps be considered as a population group skewed
towards the middle-classes.

Buying power is not all though. The nature of the Internet is such that its users will
tend to organise themselves into far more focussed groupings. Savvy marketers who
know where to look can quite easily find access to the niche markets they wish to
target. Marketing messages are most effective when they are presented directly to
the audience most likely to be interested. The Internet creates the perfect
environment for niche marketing to targeted groups.

6. Adapting and closed loop marketing: Closed Loop Marketing requires the
constant measurement and analysis of the results of marketing initiatives. By
continuously tracking the response and effectiveness of a campaign, the marketer

Abhishek kumar yadav 520844307 Page 83


can be far more dynamic in adapting to consumers’ wants and needs.

With E-Marketing, responses can be analyzed in real-time and campaigns can be


tweaked continuously. Combined with the immediacy of the Internet as a medium, this
means that there’s minimal advertising spend wasted on less than effective campaigns.

Maximum marketing efficiency from E-Marketing creates new opportunities to seize


strategic competitive advantages.

The combination of all these factors results in an improved ROI and ultimately, more
customers, happier customers and an improved bottom line.

Q. 3 (5) What are the legal issues involved in the e-marketing?

Ans. The Internet is a growing and a continually evolving creature that will live on in
perpetuity. As such, it would be wise to ponder the e business legal and Internet
marketing ethical issues of B2B & B2C. Whatever is written and published online will be
there forever. Imagine the billions upon billions of text information pages that are and
will be stored for a long time. There is even a site where you can go Way Back to
check out archives of other websites and view pages that were created at the
beginning of their infancy. Additionally, video, films, movies, and audio in various
applications formats are also viewable.

Now, with the new wireless web mail from cell phones and other pda communication
devices, the Internet will be affecting more lives than ever before. Security and privacy
concerns along with e-business regulatory issues will become more prevalent. It will
become more difficult to figure out who you can trust online; with all the unethical,
illegal, and Internet marketing and online advertising frauds and E business email
scams.

Important E-Business Legal Issues

1. Intellectual Property

When dealing with ethics in a B2B company and B2C clients there is a major degree of
trust and responsibility that is imparted to a person or group that maintains the Web
site. It is very important from both an ethical values based e business and legal B2B
and B2C perspective to make sure that the written words and what is portrayed about a
company are factual. Because issues arise involving marketing ethics and the
importance of understanding a business for Internet marketing issues and advertising
purposes, there are potential areas for revealing trade secrets or intellectual property if
proper B2B ethical behavior is not followed.

Abhishek kumar yadav 520844307 Page 84


Email correspondence should be private and confidential. While certain individuals
might not see any harm quoting it on the Web from an email sent, it is always advisable
to get a person’s consent prior to publishing anything. While the person might give
consent, they might not realize the full implications, on line privacy issues, or impact of
having it published online. Therefore, it would be wise to consider it very carefully
before even asking for their approval.

2. Copyright on the Internet

Copyright gives authors, artists and others the right to exclude others from using their
works. Federal rights arise automatically when a protected work has been fixed in a
tangible medium such as a floppy disk or hard drive. There are however certain limits
to Copyright as under:

a. Copyright is the right to exclude, not to publish: Copyright does not give its
owners the right to sell or distribute, for example, libelous email messages.

b. Basic limits to copyright: Although email messages and web pages may enjoy
copyright protection, rights are subject to several fundamental limits. For example,
only expression is protected, not facts or ideas.

c. Fair use: Fair use is one of the most important and least clear cut, limits to
copyright. It permits some use of others’ works even without approval. But when?
Words like "fair" or "reasonable" cannot be precisely defined, but here are a few
benchmarks.

d. Uses that advance public interests such as criticism, education or scholarship


are favored particularly if little of another’s work is copied. Uses that generate
income or interfere with a copyright owner’s income are not. Fairness also means
crediting original artists or authors.

e. Licenses implied in fact: Fair use allows limited uses of another’s work without
approval, but other uses may be approved by implication. For example, when a
message is posted to a public email list, both forwarding and archiving seem to be
implicitly allowed. It is reasonable to assume that such liberties are okay if not
explicitly forbidden. However, when forwarding, archiving or, say, using part of a
prior message to respond to an earlier message, be careful not to change the
original meaning. No one implicitly authorizes another to attribute to them an
embarrassing (or worse) message they did not write!

Other miscellaneous legal issues:

There are a number of laws already enacted by developed countries & many more are

Abhishek kumar yadav 520844307 Page 85


under implementation. Some of these are discussed here:

1. Email Marketing
2. Consumers Survey Scams and Hoaxes
3. Website Issues
4. Advertising and Keywords Scams
5. Cyber squatting
6. Web Scraping & Web Spidering
7. E mail Spoofing.
8. Meta elements or Meta tags and Key words
9. Click Fraud
10. Click Wrap Agreement
11. Spamming
12. Concept of Open Data

Abhishek kumar yadav 520844307 Page 86


E-MARKETING

(MK0008)

ASSIGNMENT-2

Q. 1 (2)Explain the bases used in the e-marketing segmentation.

Ans. Marketers can base their segmentation of consumer markets in various methods like
Geographic Segments, Demographic Segments, Psychographic Segments & Behavior
Segments. They also create segments by under mentioned methods:

a. Combining bases & focus on categories (known as Geodemographics)

b. Using any combination of variables that makes sense for their industry.

c. Utilising variables that can be used to identify and reach the right people at the
right time.

d. Using any of these four bases alone or in combination plus many other variables.

e. Knowing which variables broadly identify the target segment & which simply further
describe it because marketers use identification variables to enumerate and
access the target.

1. Geographic Segments

For E-marketing segmentation, geographic location of computers is not important to


users accessing Web sites, but it is very important to organizations with an Internet
presence. Most firms target specific cities, regions, states, or countries with their product
offerings & also develop multi segment strategies based on geography.

Product distribution strategy is a driving force behind geographic segmentation.


Research has found that the following factors have been important in such strategies:

a. Companies want to reach only customers in countries where they distribute


products.

b. Firms offering services online will only sell to geographic areas where they can
provide customer service.

c. Companies examine the proportion of Net users in its selected geographic targets
before deciding to serve the Web community.

Abhishek kumar yadav 520844307 Page 87


E-marketers using geographic variables for segmentation also evaluate online markets
by region, city, urban area, and so forth. While deciding on countries as a segmentation
base, the quality of a country’s market is evaluated using additional criteria like
infrastructure variables designed to predict their ability to access and absorb information
technology viz.-computer adoption, information & Internet. For example, in 2001,
Sweden and Norway held the top two positions as the world’s dominant information
economies because of a high mobile Internet access.

Another important factor is the language used on the internet web pages. English is no
longer the language of most Web pages and online bulletin boards; a major change from
just two years ago. The top Internet languages at present are:

 English (42%),
 Japanese (9%),
 Chinese (9%),
 Spanish (7%),
 German (7%).
This has huge implications for e-marketers desiring to reach global markets via the
Internet. Until online text appears in local languages, users in those countries will not
able to participate in e-commerce or other online activities.

2. Demographic Segments

In the Internet’s early years, the typical user was a young male, college educated, with a
high income who was basically an innovator. Now, this is true in countries with low levels
of Internet adoption. The e-marketers study about the age, income, population of a
geographical area who have knowledge of internet. Occupation of the customers also
give an idea of internet uses. This has huge implications for e-commerce and
advertising, and big changes are still taking place. The blue-collar workers of any country
are the fastest growing online occupational group that are engaged in internet surfing
and they provide a good e-marketing segment.

3. Psychographic Segments

User psychographics include the following characteristics:

a. Personality: Traits (other-oriented / self-oriented) and habits,

b. Values: Deeply held convictions (religious beliefs),

c. Lifestyle: Non-product-related behavior

d. Activities: (playing sports or eating out),

Abhishek kumar yadav 520844307 Page 88


e. Interests: Attitudes and beliefs people hold.

f. Opinions:

Attitudes are internal evaluations about people, products, and other objects which can be
either positive or negative & the evaluation process occurs inside a person’s head.
Attitude segmentation can be implemented by studying the general ways that consumers
spend time. This helps e-marketers define and describe market segments so they can
better meet consumer needs & this is very important for Web page design.

4. Behavior Segments

Behavior refers to what a person physically does (calling a toll free number to order,
shopping, or purchasing a product). Product behaviors are such a vital segment
descriptor that they form an entirely separate category. Two behavioral segmentation
variables are:

a) Benefit segmentation: On the Internet, there is something for everyone: information,


entertainment, news, social meeting places, and more. Marketers form segments
based on the benefits sought by users to design products to meet those needs.* The
best way to determine benefits sought is to look at what people actually do online.
Some suggestions are to check:

 which Web sites are the most popular.


 sites report monthly on the top online properties displays the top Web site
parent companies.
Commonly used benefit segmentation variables are:
 E-mail (8 billion e-mails flying over the Internet worldwide.)
 Shop.
 Information search.
 Online stock trading and online banking. (Online banking has a high usage but
online stock trading is quite active.)
 Streamies: People who listen to online audio.
b) Usage Segments: E-marketers identify segments according to how users behave
and use the Internet & profile the segments by user characteristics, geographical
location, and so on. When users shop online, sometimes they browse aimlessly; but
sometimes they have a specific goal.

Some of the segments which are important while doing this exercise are as follows:

Home Access: 80% of home users have slow connection speeds, making large

Abhishek kumar yadav 520844307 Page 89


graphics and other files undesirable on sites frequented from home. A small but growing
number of households have more than one PC and are networking them within the
home.

Work Access: A large number of users access the Internet from work. People spend
nearly twice as much overall time online than those who access only from home. The
audiences in all countries are much more heavily male.

E-marketing strategists can use such information to target their Web site offerings.
Strategies might include special products, the language in sites, and the amount of
interactivity and multimedia possible for work users.

Industry Specific Usage Segments: Segmenting by usage vary from one business
type to the other. For example, visitors to car sites behave differently than visitors to
other e-commerce sites. Even serious car buyers tend to visit car sites only a few times.

Segmenting by Usage Occasions: Marketers identify segments based on how


consumers are using the Web at particular moments. Four key variables have been
identified for defining discrete clusters of online behavior:

 Session length – the time a user stays online.


 Time per page – the average time a user spends on each page during a session.
 Category concentration – the percentage of time a user spends at sites belonging to
the most frequented category.
 Site Familiarity – the percentage of total session time a user spends at familiar sites,
defined as those previously visited four or more times.
Q. 2 (3)Describe the eight dimensions of differentiation in e-marketing.

Ans. Differentiation is the process of adding meaningful and valued differences to distinguish
the product from the competition. The keys to differentiating online businesses are the
creation of a distinctive and superior customer experience & the development of one-to-
one relationships with consumers. The real value added by the Internet is its ability to
differentiate according to customer relationships & provide a unique experience for each
customer. Many examples of differentiated online offerings are based on the lower costs
in acquiring and retaining online customers which are then passed onto customers.

In business terms, to differentiate means to create a benefit that customers perceive as


being of greater value to them than what they can get elsewhere. It’s not enough for us
to be different a potential customer has to take note of the difference and must feel that
the difference somehow fits their need better. (Other words that mean virtually the same
thing: Competitive Advantage; Unique Selling Proposition; or Value Proposition.)

Abhishek kumar yadav 520844307 Page 90


There are a number of differentiation dimensions and strategies for their
accomplishment. A firm can differentiate along eight dimensions:

1. Price Differentiation: Differentiating on price is probably the most common and


easily understood method. However, one has to be cautious while using this strategy.
On the one hand, potential customers might expect a lower price because they
perceive the e marketing firm as having less overhead, etc. On the other hand,
cheaper prices can evoke perceptions of lower quality, a less-stable business, etc.
And if we compete on price against competitors with deeper pockets, we can price
ourselves right into bankruptcy. Hence it is better to be creative with this differentiator
by competing on something other than straight price. For example, we might offer:

 More value offer more products or services for the same price.
 Freebies accessories, companion products, free upgrades, and coupons for
future purchases.
 Free shipping, etc. convenience sells, especially when it is free!
 Discounts includes offering regular sales, coupons, etc.
2. Focus Differentiation: This is the most important method of differentiation, and in
many ways, the easiest. Once we have done that, we have an automatic advantage
over other companies because we can become more of an expert in that one field
and we can build close relationships with key customers that will be hard to duplicate.
For example, we might differentiate ourselves through:

 Location take advantage our closeness to prospective customers.


 Customer specialization be very specific about what characteristics our
customers will have. For example, racing bicycle enthusiasts or companies with a
spiritual conscience.
 Customer relationships know customers really well, form partnerships with them,
and get them to speak for us.
 Affinity relationships associate our product/service with a well-known person or
organization.
 One-stop shopping offer everything our target market needs, in our area of
expertise.
 Wide selection (within our niche) although this one may seem to be the opposite
of focus the key is to be very specific in one dimension and very broad in
another.
3. Product Differentiation: The e-marketing company can differentiate itself by

Abhishek kumar yadav 520844307 Page 91


providing a limitless assortment of products. Differentiation may include
customization, bundling and attractive pricing of products. E- marketing offers a
combination of products/services that the individual consumer needs at attractive
prices & also supports one-to-one relationship building with each customer, critical for
a company’s long-term success. Such sales may not rely as heavily on product
packaging as do traditional retailers. Consumers might require products with more
utilitarian packaging & the products will be shipped from the distributor directly to the
consumer and thus never appear on retailers’ shelves. Hence products don’t need the
expensive, colorful packaging for store display. Products only need a size and shape
that is functional and useful for the consumer. Packaging minimization will reduce
waste and costs. This can result in lower prices, or more reinvestment in higher-
quality, single-layer packaging enhancements.

The key to successful differentiation in this category, again, is to know the customers,
really, really well. For example, the product or service could stand out in one of these
ways:

 Quality- create a product or service that is exceptional in one or more ways.


Examples: Lasts longer
 Better
 Easier to use
 Safer
 New/First be the first one to offer something in the location/field.
 Features/Options offer lots of choices, unusual combinations, or solve a problem
for a customer in a way no one else does.
 Customization – the firm may be able to more easily handle special orders than
big, mass-market competitors.
4. Service Differentiation: Customer service can be enhanced by 24 hour customer
feedback through e-mail & also the ability to respond more rapidly to customer
concerns. Home delivery of groceries and online banking and securities trading are
becoming increasingly popular. They are differentiated both by the features they offer
and the service consumption experiences. Excellent customer service is a great
opportunity for differentiation and another natural advantage for firms that already
know what’s important to their customers.

The firm should build the reputation on making customers feel really good about
doing business with them. This concept works great with referral marketing, too.
Some examples are:

Abhishek kumar yadav 520844307 Page 92


 Deliver fast , next day, or one-hour, make it faster than customers think possible.
 Unique channel – offer a service over the phone or Internet instead of in person
or in their office rather than yours.
 Service-delight customers! – it may seem expensive to offer exceptional service
but it pays off in word-of-mouth advertising.
 Before/during/after-sales support – provide technical or other support to
customers using the product. You might use joint ventures to provide that support
but customers will perceive it as being from you!
 Guarantee/warranty – offer 100% money-back, or free replacement parts.
5.Personnel Differentiation: In the past, personalized service and one-to-one
relationships required the interaction of skilled personnel. Now, the Internet allows
companies to deliver their products and services through low-cost channels that
automate the process and remove the expensive human element. Lower transaction
& marketing costs results in cost leadership advantage over offline companies & also
cost reduction for the end user & higher levels of service. But as more companies
offer products and services online, this cost advantage between online and offline
operations will gradually shrink over time.

6.Channel Differentiation: The Internet is a location-free, time-free distribution and


communication channel. The Internet serves as a transaction and distribution
channel. The Internet provides highly specialized personal services and “do it
yourself” websites. The Internet works as a distribution channel, a communication
channel & a relationship channel. Internet is the channel through which it can reach
customers, display a diversified assortment of offerings & differentiate itself.

There are multiple levels of online channel differentiation:


 Product or service information online which is an advantage over companies with
no Web presence,
 Exploits the Internet as a communication channel.
 Commercial transactions online,
 Exploits the Internets as a transaction and distribution channel.
 The differentiation of competitors’ Internet-related service offerings. For example,
in the banking industry, one bank may provide a virtual pass book facility while
another bank may provide an interactive portfolio management service to key
account holders.
7. Image Differentiation: A company can differentiate itself by creating a unique

Abhishek kumar yadav 520844307 Page 93


experience called “experience branding”. Experience Branding happens when a
company differentiates itself by creating a unique customer experience, increases
customer loyalty & retention which produce referral business. By this process, firms
can greatly improve their ability to retain customers, target key customer segments
and enhance network profitability. Such differentiation has to be built upon the ability
to create huge perceptual differences from other aspects of brand positioning. The
Internet’s interactivity allows companies to respond more quickly to customer
requests. Even the computer industry which was earlier focusing on product
differentiation by computing speed is now turning to image differentiation. For
example Symantec which markets system maintenance & protection software is
fostering the image of “ambulance” for the computer industry. Differentiation has to be
built upon the ability to create huge perceptual differences from other aspects of
brand position.

8. Customer Relationship: The Internet has made pricing information widely available
to suppliers, customers, and competitors. This reduces price differences between
suppliers, reduces the importance of price competition and increases the importance
of differentiation. Hence it is important to create brand loyalty and use the Internet to
build one-to-one relationships with customers.

CRM (Customer Relationship Management) System helps companies in setting up a


frontline information system for sharing information about the customers across all
interface units. eCRM is nothing but the electronic counter part of CRM. It doesn’t
replace the traditional channels of communication, such as phone or fax, but is just
another extension for the customers to interact with companies on a one-to-one
basis. It’s a more personalized and interactive form of communication and
synchronizes communications across both electronic as well as traditional channels.
Such a system has training benefits too. A new salesperson, for instance, can use the
CRM package to quickly learn about his customers or their past history.

Q. 3 (6) Discuss the building blocks of E-CRM.

Ans. Customer relationship management (CRM) is a broad term that covers concepts used by
companies to manage their relationships with customers, including the capture, storage
and analysis of customer, vendor, partner, and internal process information. A firm using
relationship marketing focuses more on wallet share than on market share.

e-CRM is the electronically delivered or managed subset of CRM. The user of an e-CRM
solution uses the resources of the internet or other digital media to deliver elements of a
marketing relationship with the customer. At its most sophisticated, eCRM can be
defined as an enterprise-wide technology driven approach to the management of all data
relating directly or indirectly to customers, in turn helping to define overall product

Abhishek kumar yadav 520844307 Page 94


development and marketing strategies. The term eCRM is also be used to describe the
customer-facing, digitally-delivered portion of CRM.

The Gartner Group model of e-CRM covers following 8 building blocks:

1. E-CRM Vision: To be successful, the CRM vision must start at the top and filter
throughout the company to keep the firm customer focused. One key aspect of CRM
vision is how to guard customer privacy. The benefits of using customer data must be
balanced by the need to satisfy customers and not anger them. TRUSTe provides its
seal and logo to any Web site meeting its privacy philosophies.

2. E-CRM Strategy: E-marketers must determine their objectives and strategies before
buying CRM technology. Many CRM goals refer to customer loyalty. An important
CRM strategy is to move customers up the relationship intensity pyramid.

3. Valued Customer Experience: Consumers are constantly bombarded by marketing


communications and unlimited product choices. According to Jagdish Sheth (1995),
the basic tenet of CRM is choice reduction. Many consumers are “loyalty prone,” and
will stick with the right product as long as its promises are fulfilled. Synchronous and
asynchronous technologies can provide automated and human services that solve
customer problems.

4. Organizational Collaboration: Marketers collaborate within and outside the


organization to focus on customer satisfaction. CRM, or “front-end” operations, can
be linked with the entire supply chain management system (SCM), or “back-end”
operations. Customer service reps have access to inventories. Producers and
wholesalers constantly receive data that can be utilized for production and delivery.
The use of extranets, two or more intranet networks that share information, allows
CRM-SCM integration.

5. E-CRM Processes: Firms use specific processes to move customers through the
customer care life cycle. E-CRM processes are used to:

 Identify customers.
 Differentiate customers.
 Customize the marketing mix.
 Interact with customers.
Firms can identify high value customers by mining customer databases and profiling
customers in terms of:
 Recency of purchases.

Abhishek kumar yadav 520844307 Page 95


 Frequency of purchases.
 Monetary value of purchases.
6. E-CRM Information: The more information a firm has, the better value it can provide
to each current or prospective customer. Firms gain much information by tracking
behavior electronically, by using Bar code scanner data & also through software that
tracks online movement, time spent per page, and purchase behavior.

7. E-CRM Technology: Technology greatly enhances CRM processes. Firms use


company-side tools to push customized information to users. Client-side tools allow
the customer to pull information that initiates the customized response from the firm.
Company-side Tools include interactive point-of-sale terminals which are located on a
retailer’s counter and used to capture data and present targeted communication. A
firm may also listen to users and build community by providing a space for user
conversation on the Web site like Chat sites & Bulletin boards. The following are
some examples of how technology can be used for building up CRM.

Company side tools: Company side tools include the following:

a. e-mail databases: Marketers use e-mail databases to build relationships by


keeping in touch with useful and timely information. E-mail can be sent to
individuals or sent en masse using a distributed e-mail list.

b. Collaborative filtering: Collaborative filtering software gathers opinions of like-


minded users and returns those opinions to the individual in real time.

c. Real-time profiling: Real-time profiling occurs when special software tracks a


user’s movements through a Web site, then compiles and reports on the data at
a moment’s notice.

d. Data mining: Data mining involves the extraction of hidden predictive information
in large databases through statistical analysis.

e. Web log analysis: Every time a user accesses a Web site, the visit is recorded
in the Web server’s log file. This file keeps track of which pages the user visits,
how long he stays, and whether he purchases or not.

f. Cookies: Cookies are small files written to the user’s hard drive after visiting a
Web site. When the user returns to the site, the company’s server looks for the
cookie file and uses it to personalize the site.

Client-side Tools: Some examples of Client side tools are as under:

Abhishek kumar yadav 520844307 Page 96


a. E-mail queries, complaints, or compliments initiated by customers or prospects
comprise incoming e-mail, and is the fodder for customer service.
b. Incoming e-mail
c. With fax-on-demand, customers telephone a firm, listen to an automated voice
menu, and select options to request a fax be sent on a particular topic.
d. Fax-on-demand
e. Web form (or HTML form) is the technical term for a form on a Web page that has
designated places for the user to type information for submission.
f. Web forms
g. Wireless Web portals send data to customer cell phones, pagers, and PDAs,
such as the PalmPilot.
h. Wireless data services
i. Personalized Web pages users easily configure at Web sites such as MyYahoo!
and many others.
j. Individualized Web portals
k. Programs that perform functions on behalf of the user, such as search engines
and shopping agents.
8. E-CRM Metrics: E-marketers use numerous metrics to assess the Internet’s value in
delivering CRM performance. For example:

 ROI
 Cost savings
 Revenues
 Customer satisfaction
One research study named customer retention, ROI and customer lift (increased
response or transaction rates) as the most important metrics. One very important
CRM metric is customer lifetime value (LTV).

 The LTV calculation demonstrates the benefits of retaining customers over time
and the need for building wallet share.

 LTV also illustrates that no matter how good customer retention is, the firm must
still focus on customer acquisition activities.

Abhishek kumar yadav 520844307 Page 97


INTERNATIONAL MARKETING

(MK-0009)

ASSIGNMENT-1

Abhishek kumar yadav 520844307 Page 98


Q. 1 Distinguish between international marketing, multinational marketing and global
marketing.

Ans If we talk in general, both the terms Global and International marketing are used
interchangeably nowadays. But if we are talking in terms of marketing theories,
International Marketing was a stage in the evolution of Global Marketing. Let us see how,

Stage 1 : Domestic Marketing : Companies manufacturing products and selling those


within the country itself. So, no international phenomenon at all.

Stage 2 : Export Marketing : Company starts exporting products to another countries


also. This is the very basic stage of global marketing. Approach of marketer in this stage is
said to be ‘ethnocentric’ because although he is selling goods to foreign countries, product
development is totally based upon the taste of local customer. So, focus is still on domestic
market

Stage 3 : International Marketing : Now, company starts selling products to various


countries and the approach is ‘Polycentric’ i.e. making different products for different
countries.

Stage 4 : Multinational Marketing : Now, in this stage, the number of countries in which
the company is doing business gets bigger than that in earlier stage. And so, instead of
producing different goods for different countries, company tries to identify different regions
for which it can deliver same product. So, same product for countries lying in one region
but different from product offered in countries of another region. e.g. a company may
decide to offer same product to India, Sri lanka and Pakistan if it thinks the taste of people
of these countries is same but at the same time offering different product for American
countries. This approach is called ‘Regiocentric approach’.

Stage 5 : Global Marketing : This is the final stage of evolution. In this stage company
really operates in a very large number of countries and for the purpose of achieving cost
efficiencies it analyses the requirement and taste of customers of all the countries and
come out with a single product which can satisfy the needs of all. This approach is called
‘Geocentric approach’.

So, one main difference between International and the Global marketing is the approach of
marketer. A truly global company instead of offering different products to different countries
(as in International Marketing), develops and offers a single product to the world.

We should note one more very interesting fact that in the early stage of Export marketing
also, the company was offering a single product to each of the countries as in the final
stage of Global marketing. But it was entirely different because in export marketing you
produce according to taste of your country and force that on other countries but in Global
approach, you take care of entire countries and develop a product which can satisfy the
need of all.

The distinguishing factors between international marketing, multinational marketing and


global marketing are tabled as below:

INTERANTIONAL MAKETING MULTINATIONAL GLOBAL


Abhishek kumar yadav 520844307 MARKETING Page 99
MARKETING
If the exporting departments are At the multi-national When a company
INTERNATIONAL MARKETING

(MK-0009)

ASSIGNMENT-2

Q. 1 Describe product standardization versus adaptation in international marketing.

Ans A product can be defined as a collection of physical, service and symbolic attributes which
. yield satisfaction or benefits to a user or buyer. A product is a combination of physical
attributes say, size and shape; and subjective attributes say image or "quality". A customer
purchases on both dimensions.

A product’s physical properties are characterized the same the world over. They can be
convenience or shopping goods or durables and non-durables; however, one can classify
products according to their degree of potential for global marketing:

a. Local products – seen as only suitable in one single market.


b. International products – seen as having extension potential into other markets.
c. Multinational products – products adapted to the perceived unique characteristics of
national markets.
d. Global products – products designed to meet global segments.
Quality, method of operation or use and maintenance (if necessary) are the catchwords in
international marketing. A failure to maintain these will lead to consumer dissatisfaction.

Changes in design are largely dictated by whether they would improve the prospects of
greater sales, and this, over the accompanying costs. Changes in design are also subject
to cultural pressures. The more culture-bound the product is, for example food, the more
adaptation is necessary. Most products fall in between the spectrum of "standardization" to
"adaptation" extremes.

Abhishek kumar yadav 520844307 Page 100


Standardization: The factors encouraging standardization are:
a. Economies of scale in production and marketing
b. Consumer mobility – the more consumers travel, the more is the demand
c. Technology
d. Image, for example "japanese", "made in".
The latter can be a factor both to aid or to hinder global marketing development.
Nagashima1 (1977) found the "made in USA" image has lost ground to the "made in
Japan" image. In some cases "foreign made" gives advantage over domestic products. In
Zimbabwe one sees many advertisements for "imported", which gives the product
advertised a perceived advantage over domestic products. Often a price premium is
charged to reinforce the "imported means quality" image. If the foreign source is negative
in effect, attempts are made to disguise or hide the fact through, say, packaging or
labelling. Mexicans are loathe to take products from Brazil. By putting a "made in
elsewhere" label on the product this can be overcome, provided the products are
manufactured elsewhere, even though its company may be Brazilian.

On the down side, there may also be significant differences in desires between cultures
and physical environments – e.g., software sold in the U.S. and Europe will often utter a
“beep” to alert the user when a mistake has been made; however, in Asia, where office
workers are often seated closely together, this could cause embarrassment.

Adaptation: The factors encouraging adaptation are:

a. Differing usage conditions – These may be due to climate, skills, level of literacy,
culture or physical conditions. Maize, for example, would never sell in Europe rolled and
milled as in Africa. It is only eaten whole, on or off the cob. In Zimbabwe, kapenta fish
can be used as a relish, but will always be eaten as a "starter" to a meal in the
developed countries.

b. General market factors – incomes, tastes etc. Canned asparagus may be very
affordable in the developed world, but may not sell well in the developing world.

c. Government – taxation, import quotas, non tariff barriers, labelling, health


requirements. Non-tariff barriers are an attempt, despite their supposed impartiality, at
restricting or eliminating competition. A good example of this is the Florida tomato
growers, who successfully got the US Department of Agriculture to issue regulations
establishing a minimum size of tomatoes marketed in the United States. The effect of
this was to eliminate the Mexican tomato industry, which grew a tomato that fell under
the minimum size specified. Some non-tariff barriers may be legitimate attempts to
protect the consumer, for example, the ever stricter restrictions on horticultural produce

Abhishek kumar yadav 520844307 Page 101


insecticides and pesticides use may cause African growers a headache, but they are
deemed to be for the public good.

d. History – Sometimes, as a result of colonialism, production facilities have been


established overseas. Eastern and Southern Africa is littered with examples. In Kenya,
the tea industry is a colonial legacy, as is the sugar industry of Zimbabwe and the
coffee industry of Malawi. These facilities have long been adapted to local conditions.

e. Financial considerations – In order to maximize sales or profits, the organization may


have no choice but to adapt its products to local conditions.

f. Pressure – Sometimes, as in the case of the EU, suppliers are forced to adapt to the
rules and regulations imposed on them if they wish to enter into the market.

Forms Of Adaptation: Adaptations can also come in several forms.

a. Mandatory adaptations: Mandatory adaptations involve changes that have to be made


before the product can be used – e.g., appliances made for the U.S. and Europe must
run on different voltages, and a major problem was experienced in the European Union
when hoses for restaurant frying machines could not simultaneously meet the legal
requirements of different countries.

b. Discretionary changes: These changes are changes that do not have to be made
before a product can be introduced (e.g., there is nothing to prevent an American firm
from introducing an overly sweet soft drink into the Japanese market), although
products may face poor sales if such changes are not made. Discretionary changes
may also involve cultural adaptations – e.g., in Sesame Street, the Big Bird became the
Big Camel in Saudi Arabia.

c. Physical product vs. communication adaptations: In order for gasoline to be


effective in high altitude regions, its octane must be higher, but it can be promoted
much the same way. On the other hand, while the same bicycle might be sold in China
and the U.S., it might be positioned as a serious means of transportation in the former
and as a recreational tool in the latter. In some cases, products may not need to be
adapted in either way (e.g., industrial equipment), while in other cases, it might have to
be adapted in both (e.g., greeting cards, where the occasions, language, and
motivations for sending differ).

d. Lack of equivalent at home: Finally, a market may exist abroad for a product which
has no analogue at home – e.g., hand-powered washing machines.

There are ten basic criteria for adaptation that all who wish to deal with individuals, firms,

Abhishek kumar yadav 520844307 Page 102


or authorities in foreign countries should be able to meet. They are:

1. open tolerance,
2. flexibility,
3. humility,
4. justice/fairness,
5. ability to adjust to varying tempos,
6. curiosity/interest,
7. knowledge of the country,
8. liking for others,
9. ability to command respect, and
10. ability to integrate oneself into the environment.
Q. 2 What is direct marketing? Give example of any MNC practicing direct marketing in
India. What are the advantages of direct marketing?

Ans Direct marketing is a sub-discipline and type of marketing. There are two main definitional
. characteristics which distinguish it from other types of marketing.

 The first is that it attempts to send its messages directly to consumers, without the use
of intervening media. This involves commercial communication (direct mail, e-mail,
telemarketing) with consumers or businesses, usually unsolicited.

 The second characteristic is that it is focused on driving a specific "call-to-action." This


aspect of direct marketing involves an emphasis on trackable, measurable positive
(but not negative) responses from consumers (known simply as "response" in the
industry) regardless of medium.

Direct marketing is a system of marketing that integrates ordinarily separate marketing mix
elements to sell directly to both consumers and other businesses, bypassing retail stores
and personal sales calls. It is used by virtually every consumer and business to-business
category, from banks to airlines to nonprofit organizations. Because the customer
responds directly to the company making the offer, international considerations that apply
to communications, distribution, and sales have to be considered. Direct marketing uses a
wide spectrum of media, including direct mail; telephone; broadcast, including television
and radio; and print, including newspapers and magazines.

Direct marketing is a sometimes controversial sales method by which advertisers approach


potential customers directly with products or services. The most common forms of direct
marketing are:

 Telephone sales, solicited or unsolicited emails, catalogs, leaflets, brochures and

Abhishek kumar yadav 520844307 Page 103


coupons.

 Successful direct marketing also involves compiling and maintaining a large database
of personal information about potential customers and clients. These databases are
often sold or shared with other direct marketing companies.

The different channels of direct marketing are:

Direct Mail: The most common form of direct marketing is direct mail sometimes called
junk mail, used by advertisers who send paper mail to all postal customers in an area
or to all customers on a list.

Telemarketing : The second most common form of direct marketing is telemarketing in


which marketers contact consumers by phone.

Email Marketing: Email Marketing may have passed telemarketing in frequency at this
point and is a third type of direct marketing. A major concern is spam, which actually
predates legitimate email marketing.

Door To Door Leaflet Marketing: Leaflet Distribution services are used extensively by
the fast food industries, and many other business focusing on a local catchment
Business to consumer business model, similar to direct mail marketing.

Broadcast Faxing : A fourth type of direct marketing, broadcast faxing, is now less
common than the other forms. This is partly due to laws in the United States and
elsewhere which make it illegal.

Voicemail Marketing: A fifth type of direct marketing has emerged out of the market
prevalence of personal voice mailboxes, and business voicemail systems.

Direct Response Television Marketing : Direct marketing on TV (commonly referred to


as DRTV) has two basic forms: long form (usually half-hour or hour-long segments
that explain a product in detail and are commonly referred to as infomercials) and
short form which refers to typical 0:30 second or 0:60 second commercials that ask
viewers for an immediate response (typically to call a phone number on screen or go
to a website).

Direct Selling : Direct selling is the sale of products by face-to-face contact with the
customer, either by having salespeople approach potential customers in person, or
through indirect means such as Tupperware parties.

Advantages Of Direct Marketing:

1. For many companies or service providers with a specific market, the traditional forms of

Abhishek kumar yadav 520844307 Page 104


advertising (radio, newspapers, television, etc.) may not be the best use of their
promotional budgets. For example, a company which sells a hair loss prevention
product would have to find a radio station whose format appealed to older male
listeners who might be experiencing this problem. There would be no guarantee that
this group would be listening to that particular station at the exact time the company's
ads were broadcast. Money spent on a radio spot (or television commercial or
newspaper ad) may or may not reach the type of consumer who would be interested in
a hair restoring product.

2. This is where direct marketing becomes very appealing. Instead of investing in a


scattershot means of advertising, companies with a specific type of potential customer
can send out literature directly to a list of pre-screened individuals.

3. Direct marketing firms may also keep email addresses of those who match a certain
age group or income level or special interest. Manufacturers of a new dog shampoo
might benefit from having the phone numbers and mailing addresses of pet store
owners or dog show participants. Direct marketing works best when the recipients
accept the fact that their personal information might be used for this purpose. Some
customers prefer to receive targeted catalogs which offer more variety than a general
mailing.

4. There are certain circumstances when direct marketing may be more useful – e.g.,
when absolute margins are very large (e.g., computers) or when a large inventory may
be needed (e.g., computer CDs) or when the customer base is widely dispersed (e.g.,
bee keepers).

5. Direct marketing offers exceptional opportunities for segmentation because marketers


can buy lists of consumer names, addresses, and phone-numbers that indicate their
specific interests. For example, if we want to target auto enthusiasts, we can buy lists of
subscribers to auto magazines and people who have bought auto supplies through the
mail. We can also buy lists of people who have particular auto makes registered.

MNC’s Practicing Direct Marketing In India:

Multinational companies are the organizations or enterprises that manage production or


offer services in more than one country. And India has been the home to a number of
multinational companies. In fact, since the financial liberalization in the country in 1991, the
number of multinational companies in India has increased noticeably. Though majority of
the multinational companies in India are from the U.S., however one can also find
companies from other countries as well.

There are a number of reasons why the multinational companies are coming down to India.

Abhishek kumar yadav 520844307 Page 105


India has got a huge market. It has also got one of the fastest growing economies in the
world. Besides, the policy of the government towards FDI has also played a major role in
attracting the multinational companies in India.

For quite a long time, India had a restrictive policy in terms of foreign direct investment. As
a result, there was lesser number of companies that showed interest in investing in Indian
market. However, the scenario changed during the financial liberalization of the country,
especially after 1991. Government, nowadays, makes continuous efforts to attract foreign
investments by relaxing many of its policies. As a result, a number of multinational
companies have shown interest in Indian market.

List Of MNC’s Doing Direct Marketing In India:

The list of multinational companies in India is ever-growing as a number of MNCs are


coming down to this country now and then. Following are some of the major multinational
companies operating their businesses by Direct Marketing in India:

a. British Petroleum
b. Vodafone
c. Ford Motors
d. LG
e. Samsung
f. Hyundai
g. Accenture
h. Reebok
i. Skoda Motors
j. ABN Amro Bank
k. Pepsi
l. Coco- Cola
A majority of multinational corporations operating in India have rated the country as a
better investment destination on various parameters than China, Brazil, the UK, France,
Australia and Singapore.

India scores over China on skilled labour force and its market growth prospects are
brighter than the UK and Singapore, according to a perception survey conducted by
Federation of Indian Chambers of Commerce & Industry (Ficci) among 135 MNCs in India.
These include

Procter & Gamble,


Nestle,

Abhishek kumar yadav 520844307 Page 106


Hyundai,
Ford
Colgate Palmolive.
Q. 3 What are various challenges that marketer faces while doing international
advertising?

Ans When organizations advertise across international boundaries, a number of important


. factors have to be taken into consideration. Whilst the process is ostensibly
straightforward, that is someone (seller) says something (message) to someone (buyer)
through a medium) the process is compounded by certain factors.

International advertising entails dissemination of a commercial message to target


audiences in more than one country. Target audiences differ from country to country in
terms of how they perceive or interpret symbols or stimuli; respond to humor or emotional
appeals, as well as in levels of literacy and languages spoken. How the advertising
function is organized also varies. In some cases, multinational firms centralize advertising
decisions and budgets and use the same or a limited number of agencies worldwide.

International advertising can, therefore, be viewed as a communication process that takes


place in multiple cultures that differ in terms of values, communication styles, and
consumption patterns. International advertising is also a business activity involving
advertisers and the advertising agencies that create ads and buy media in different
countries. The sum total of these activities constitutes a worldwide industry that is growing
in importance. International advertising is also a major force that both reflects social
values, and propagates certain values worldwide.

Various challenges that marketer faces while doing international advertising are:

1. The advertising paradigm : These mitigating factors can be called "noise" and have
an effect on the decision to "extend", "adapt" or "create" new messages. Language
differences may mean that straight translation is not enough when it comes to message
design. Advertising may also play different roles within developed, between developed
and underdeveloped and within underdeveloped countries. In developing countries
"education" and "information" may be paramount objectives. In developed countries,
the objectives may be more persuasive.

Cultural differences may account for the greatest challenge. However, many experts
challenge the need to adapt messages and images, as they argue that consumer
differences between countries are diminishing. Changes may be needed only in
translation. However, this is only one point of view, as there is no doubt that cultural
differences do exist across the world. For example, it would be quite unacceptable to

Abhishek kumar yadav 520844307 Page 107


have swim suited ladies advertising sun care products in Muslim countries.

Three major difficulties occur in attempting to communicate internationally:

• The message may not get through to the intended recipient, due to lack of media
knowledge;
• The message may get through but not be understood, due to lack of audience
understanding and:
• The message may get through, be understood but not provoke action. This may be
due to lack of cultural understanding.
2. Campaign design: Before embarking on a promotion campaign, the following
questions, among many others, must be answered.

 What can be said about the product?


 Which audience is being reached?
 What resources does the organization have?
 Can someone do it better, say an agency?
 Basic steps in conducting an advertising campaign:
3. Objectives : Advertising must only be undertaken for a specific purpose(s) and this
purpose must be translated into objectives. Whilst difficult to directly attribute to
advertising, persuasive advertising’s ultimate objective is to obtain sales. Other
objectives include building a favorable image, information giving, stimulating distributors
or building confidence in a product. Whatever objective(s) are pursued, these must be
related to the product life cycle and the stage the product is in.

4. Budgets: Budgets can be set in a variety of ways. Many budgets use a percentage of
past or future sales, objective and task methods, or rule of thumb. "Scientific" methods
include sales response methods and linear programming.

5. Agency: Agencies can be used or not, depending on the organizations’ own abilities,
confidence in the market and market coverage. Many organizations, like Lintas and
Interpublic, are worldwide and offer a wide range of expertise.

6. Message selection: Message selection is probably where the most care has to be
taken. Decisions hinge on the standardization or adaptation of message decision,
language nuances and the development of global segments and customers. Message
design has three elements, illustration, layout and copy.

Advertising appeals should be consistent with tastes, wants and attitudes in the market.

Abhishek kumar yadav 520844307 Page 108


Coke and Pepsi have found universal appeal. With the "post-modern age" now affecting
marketing, message design is becoming particularly crucial. It is not just a question of
selling, but of crafting images. It is often the image, not the product, which is
commercialized. Products do not project images, products fill the images which the
communication campaign projects. Coke’s "Life" theme is a classic in this regard.

In illustrations and artwork, some forms are universally understood. Coke, again, with
its "life" theme is applicable anywhere. Cheese and beer advertisements would go well
together in Germany, but it would have to be cheese and wine in France.

Copy, or text, has been the subject of much debate. Effective translation requires good
technical knowledge of the original and translated language, the product and the
objectives of the original copy. Care has to be taken that the meaning does not get lost
in translation.

7. Media selection : There is a great difference in variety and availability of media across
the world. The choice of media depends on its cost, coverage, availability, character
(national or local or international) and its "atmosphere".

In advertising the choice is television, radio, press, magazines, cinema, posters, direct
mail, transport and video promotion. In promotion the choice is wide between money-off
offers, discounts, extra quantities, and so on. Other forms of promotion include
exhibitions, trade missions, public relations, selling, packaging, branding and sponsored
events. Governments can be a very powerful promotion source, both by providing
organizations like Horticultural Promotion Councils and by giving information and
finance.

8. Campaign scheduling : Scheduling international campaigns is difficult, especially if


handled alone rather than with an agency or third party. Scheduling decisions involve
decisions on when to break the campaign, the use of media solely or in combination,
and the specific dates and times for advertisements to appear in the media.

9. Evaluation : Advertising campaign evaluation is not very easy at the best of times.
Whilst it would be nice to say that "X" sales had resulted from "Y" advertising inputs, too
many intervening factors make the simple tie-up difficult. Evaluation takes place at two
levels – the effectiveness of the message and the effectiveness of the media. Few
African developing countries, except Kenya, have any sophisticated methods for
campaign evaluation. Measures include message recall tests, diary completion, and
brand recall.

10. Organization and control : Whilst companies like Nestle may have centrally
organized and controlled advertising campaigns, many are devolved to local

Abhishek kumar yadav 520844307 Page 109


subsidiaries or agencies. The degree of autonomy afforded to local subsidiaries
depends on the philosophy of the organization and the relative knowledge of the local
market by the principal.

Abhishek kumar yadav 520844307 Page 110

You might also like